mesmerizers.files.wordpress.com  · web viewq. explain concept of balance scorecard. ans: the...

68
Q. Explain concept of Balance Scorecard. Ans: The Balanced Scorecard (BSC) is a performance management tool which began as a concept for measuring whether the smaller-scale operational activities of a company are aligned with its larger-scale objectives in terms of vision and strategy. By focusing not only on financial outcomes but also on the operational, marketing and developmental inputs to these, the Balanced Scorecard helps provide a more comprehensive view of a business, which in turn helps organizations act in their best long-term interests. Organizations were encouraged to measure, in addition to financial outputs, those factors which influenced the financial outputs. For example, process performance, market share / penetration, long term learning and skills development, and so on. The underlying rationale is that organizations cannot directly influence financial outcomes, as these are "lag" measures, and that the use of financial measures alone to inform the strategic control of the firm is unwise. Organizations should instead also measure those areas where direct management intervention is possible. In so doing, the early versions of the Balanced Scorecard helped organizations achieve a degree of "balance" in selection of performance measures. In practice, early Scorecards achieved this balance by encouraging managers to select measures from three additional categories or perspectives: "Customer," "Internal Business Processes" and "Learning and Growth." Use-Implementing Balanced Scorecards typically includes four processes: 1. Translating the vision into operational goals; 2. Communicating the vision and link it to individual performance; 3. Business planning; index Setting 4. Feedback and learning, and adjusting the strategy accordingly. The Balanced Scorecard is a framework, or what can be best characterized as a “strategic management system” that claims to incorporate all quantitative and abstract measures of true importance to the enterprise. According to Kaplan and Norton, “The Balanced Scorecard provides managers with the instrumentation they need to navigate to future competitive success”. Many books and articles referring to Balanced Scorecards confuse the design process elements and the Balanced Scorecard itself. In particular, it is common for people to refer to a “strategic linkage model” or “strategy map” as being a Balanced Scorecard. Although it helps focus managers' attention on strategic issues and the management of the implementation of strategy, it is important to remember that the Balanced Scorecard itself has no role in the formation of strategy. In fact, Balanced Scorecards can comfortably co- exist with strategic planning systems and other tools.

Upload: others

Post on 17-Mar-2020

1 views

Category:

Documents


0 download

TRANSCRIPT

Page 1: mesmerizers.files.wordpress.com  · Web viewQ. Explain concept of Balance Scorecard. Ans: The Balanced Scorecard (BSC) is a performance management tool which began as a concept for

Q. Explain concept of Balance Scorecard.Ans: The Balanced Scorecard (BSC) is a performance management tool which began as a concept for measuring whether the smaller-scale operational activities of a company are aligned with its larger-scale objectives in terms of vision and strategy. By focusing not only on financial outcomes but also on the operational, marketing and developmental inputs to these, the Balanced Scorecard helps provide a more comprehensive view of a business, which in turn helps organizations act in their best long-term interests.Organizations were encouraged to measure, in addition to financial outputs, those factors which influenced the financial outputs. For example, process performance, market share / penetration, long term learning and skills development, and so on. The underlying rationale is that organizations cannot directly influence financial outcomes, as these are "lag" measures, and that the use of financial measures alone to inform the strategic control of the firm is unwise. Organizations should instead also measure those areas where direct management intervention is possible. In so doing, the early versions of the Balanced Scorecard helped organizations achieve a degree of "balance" in selection of performance measures. In practice, early Scorecards achieved this balance by encouraging managers to select measures from three additional categories or perspectives: "Customer," "Internal Business Processes" and "Learning and Growth."Use-Implementing Balanced Scorecards typically includes four processes:

1. Translating the vision into operational goals; 2. Communicating the vision and link it to individual performance; 3. Business planning; index Setting 4. Feedback and learning, and adjusting the strategy accordingly.

The Balanced Scorecard is a framework, or what can be best characterized as a “strategic management system” that claims to incorporate all quantitative and abstract measures of true importance to the enterprise. According to Kaplan and Norton, “The Balanced Scorecard provides managers with the instrumentation they need to navigate to future competitive success”.Many books and articles referring to Balanced Scorecards confuse the design process elements and the Balanced Scorecard itself. In particular, it is common for people to refer to a “strategic linkage model” or “strategy map” as being a Balanced Scorecard. Although it helps focus managers' attention on strategic issues and the management of the implementation of strategy, it is important to remember that the Balanced Scorecard itself has no role in the formation of strategy. In fact, Balanced Scorecards can comfortably co-exist with strategic planning systems and other tools.

Q. How balanced scorecard is implemented?Ans. The grouping of performance measures in general categories (perspectives) is seen to aid in the gathering and selection of the appropriate performance measures for the enterprise.

Process of Balance Score CardThe first part of the process is creating a model for the scorecard. First, review and clarify strategies; this often requires some facilitated arguments and discussions, so that broad disagreements can be dealt with. Many organizations do not adequately resolve their strategic differences, so people work separately towards different visions. For example, one automaker's strategies for selling cars were split by group: the CEO believed in forming alliances with exotic makers, the sales executive leaned on rebates, and the

Page 2: mesmerizers.files.wordpress.com  · Web viewQ. Explain concept of Balance Scorecard. Ans: The Balanced Scorecard (BSC) is a performance management tool which began as a concept for

product group, with limited budgets, souped up existing economy cars. The result was an ineffectual, expensive hodgepodge. When one leader with a clear vision worked with others to develop strategies, they were able to transform the culture and organizational structure to produce vehicles that eventually saved the company. The next step is agreeing on what capabilities are needed within the company to actually pursue the strategy. At the automaker, they needed to innovate without access to capital. They created a new design process that included as many people as possible, from suppliers to factory workers and mechanics, so that everyone shared the same strategic goals, and worked together to pursue them. Lead time was cut in half, costs were slashed, and the products gained immediate critical acclaim; sales went up as costs went down. As if to show that this was not simply an issue of new technology, the same automaker changed their leadership style and methods, abandoning their principles of involvement - and suffered higher costs and lower sales.

The Four perspectives of Balance Score CardThe scorecard measures an organization’s performance from four key perspectives: The grouping of performance measures in general categories (perspectives) is seen to aid in the gathering and selection of the appropriate performance measures for the enterprise. Four general perspectives have been proposed by the Balanced Scorecard:1) Financial Perspective:The financial perspective examines if the company’s implementation and execution of its strategy are contributing to the bottom-line improvement of the company. It represents the long-term strategic objectives of the organization and thus it incorporates the tangible outcomes of the strategy in traditional financial terms. 2) Customer Perspective:The customer perspective defines the value proposition that the organization will apply to satisfy customers and thus generate more sales to the most desired customer groups. The measures that are selected for the customer perspective should measure both the value that is delivered to the customer which may involve time, quality, performance and service and cost and the outcomes that come as a result of this value proposition. The value proposition can be centered on one of the three: operational excellence, customer intimacy or product leadership, while maintaining threshold levels at the other two.3) Internal process perspective:The internal process perspective is concerned with the processes that create and deliver the customer value proposition. It focuses on all the activities and key processes required in order for the company to excel at providing the value expected by the customers both productively and efficiently. These can include both short-term and long-term objectives as well as incorporating innovative process development in order to stimulate improveme4) Innovation and learning Perspective:The innovation and learning perspective is the foundation of any strategy and focuses on the intangible assets of an organization, mainly on the internal skills and capabilities that are required to support the value-creating internal processes. The Innovation & Learning Perspective is concerned with the jobs (human capital), the systems (information capital), and the climate (organization capital) of the enterprise.

Page 3: mesmerizers.files.wordpress.com  · Web viewQ. Explain concept of Balance Scorecard. Ans: The Balanced Scorecard (BSC) is a performance management tool which began as a concept for

These three factors relate to what Kaplan and Norton claim is the infrastructure that is needed in order to enable ambitious objectives in the other three perspectives to be achieved.

Q. What is balance scorecard? Describe process of implementation and possible difficulties?Ans: The Balanced scorecard is an example of a performance measurement system. The business units are

to be assigned goals and then measured from the following 4 perspectives:1. Financial ( e.g. Profit margins, return on assets, cash flow)2. Customer (e.g. market share, customer satisfaction index)3. Internal business (e.g. employee retention, cycle time reduction)4. innovation and learning (e.g. percentage of sales from new products)

The balanced scorecard fosters a balance among different strategic measures in an effort to achieve goal congruence, thus encouraging employees to act in the organizations best interest.It is a tool that helps the company focus, improves communication, sets organizational objectives and provides feedback on strategy.Every measure on a balanced scorecard addresses an aspect of a company’s strategy. In creating the balanced scorecard, executives must choose a mix of measurements that

1. Accurately reflect the critical factors that will determine the success of company’s strategy.2. Show the relationships among individual measures in a cause and effect manner, indicating how

non financial measures affect long term financial results.3. Provide a broad based view of the current status of the company.

Implementing a performance measurement system:Implementation of a performance measurement system involves 4 general steps:

1. Define strategy2. Define measures of strategy3. integrate measures into the management system4. Review measures and results frequently

Each of these steps is iterative requiring the participation of senior executives and employees throughout the organization.

1. Define Strategy :The scorecard builds a link between strategy and operational action. Therefore the process of defining a scorecard begins by defining the organizational strategy. In this phase, it is important that organizational goals are explicit and that targets have been developed.For a single industry firm (e.g. Analog Devices, Wrigley), the scorecard should be developed at the corporate level and then cascaded down to functional levels and below.However for multi-business firm (e.g. GE, DuPont) the scorecard should be developed at business unit level. It is important that functional departments within a business unit have their own scorecards and that the business unit scorecard and scorecards below that level be aligned.2. Define measures of strategy: The next step is to develop measures to support the articulation strategy. The organization must focus on a few critical measures at this point or management will be overloaded with measures. Also it is important that the individual measures be linked with each other in a cause effect manner.

Page 4: mesmerizers.files.wordpress.com  · Web viewQ. Explain concept of Balance Scorecard. Ans: The Balanced Scorecard (BSC) is a performance management tool which began as a concept for

3. Integrate measures into the management system: The scorecard must be integrated with the organizations formal and informal structures, culture and HR practices. For instance the effectiveness of the scorecard will be compromised if manager’s compensation is based only on financial performance.4. Review measures and results frequently: Once the scorecard is up and running it must be consistently and continually reviewed by senior management. The organization should look for the following:

How is the organization doing according to the outcome measures?How s the organization doing according to the driver’s measures?How has the organization’s strategy changed since the last review?How have the scorecard measures changed?These reviews tell the management whether the strategy is being implemented correctly and how successfully it is working.They keep measures aligned to ever changing strategies. They improve measurement.Difficulties in implementation:

1. Poor correlation between non-financial measures and results: There is no guarantee that future profitability will follow target achievements in any non financial area. This is a serious problem because there is an inherent assumption that future profitability does follow from achieving individual measures.This is a problem when we try to develop proxy measures for future performance. While it does not mean that systems with several measures should be abandoned, it is important for companies to understand that links between non-financial measures and financial performance are not well understood.

2. Fixation on financial results: Not only are most senior managers well trained and adept with financial measures they also keenly feel pressure regarding the financial performance of their companies. Shareholders are vocal and board of directors frequently apply pressure on the shareholders behalf. This pressure may overwhelm the long term, non-certain payback of non financial measures.Poorly designed incentive programs create additional pressure. Senior managers most often are compensated for financial performance. This can disrupt goal congruence, causing managers to be more concerned about financial measures than any other measure. Even companies that have tied rewards to multiple measures may have disproportionate bias towards financial performance.

3. Measures are not updated: Many companies do not have a formal mechanism for updating the measures to align with changes in strategy. As a result, the companies continue to use measures based on yesterday’s strategy. Additionally measures often build up inertia, particularly as people get comfortable using them.

4. Measurement overload: How many critical measures can one manager track at one time without losing focus? If the no. is too few, the manager is ignoring measures that are critical to monitoring strategy execution. If there are too many measures the manager may risk losing focus in trying to do too many things at once.

Page 5: mesmerizers.files.wordpress.com  · Web viewQ. Explain concept of Balance Scorecard. Ans: The Balanced Scorecard (BSC) is a performance management tool which began as a concept for

5. Difficulty in establishing trade offs: Some companies combine financial and non financial measures into a single report and give weights to individual measures. But most scorecards do not assign explicit weights across measures. Without such weights it becomes difficult to establish tradeoffs between financial and non financial measures.

Q4. What is profit centre?Ans : When the manager is held responsible for both cost (inputs) and revenues (output) and thus, for profit of a responsibility centre, it is called a Profit Centre. In a Profit Centre, both inputs and outputs are measured in terms of money. The difference between revenues and costs represents profit where the former exceeds the latter and loss when it is vice versa. The term “revenue” with reference to responsibility accounting is used in a different sense altogether. According to generally accepted principles of accounting, revenues are recognised only when sales are made to external customers. For evaluating the performance of a profit centre, the revenue represents a monetary measure of output emanating from a profit centre during a given period, irrespective of whether the revenue is realised or not. The underlying principle is that a department has output representing goods and services which are capable of monetary measurement.

The relevant profit to facilitate the evaluation of performance measurement of a profit centre is the pre-tax profit of a responsibility centre. The profit of all the departments so calculated will not necessarily be equivalent to the profit of the entire organisation. The variance will arise because costs which are not attributable to any single department, are excluded from the computation of the department's profits and the same are adjusted while determining the profits of the whole organization. Hence, it is the divisional profit which is required for the purpose of managerial control. As the profit provides more effective appraisal of the manager's performance, the manager of the profit centre is highly motivated in his decision-making relating to inputs and outputs so that profits can be maximized. In consonance with the above objective, by creating more profit centres in an organisation, decentralisation of activities can be easily effected. The profit centre approach cannot be uniformly applied to all responsibility centres. The following are the criteria to be considered for making a responsibility centre into a profit centre. A profit centre must maintain additional record keeping to measure inputs and outputs in monetary terms.When a responsibility centre renders only services to other departments at the instance of the management, e.g., internal audit.it cannot be made a profit centre. A profit centre will gain more meaning and significance only when the divisional managers of responsibility centres have empowered adequately in their decision making relating to quality and quantity of outputs and also their relation to costs. If the output of a division is fairly homogeneous (e.g., cement), a profit centre will not prove to be more beneficial than a cost centre. Again, due to intense competition prevailing among different profit centres, there will be continuous friction among the centres arresting the growth and expansion of the whole organisation. A profit centre will generate too much of interest in the short-run profit to the detriment of long-term results.

Q.Explain types of expense centre along with their sketches

Page 6: mesmerizers.files.wordpress.com  · Web viewQ. Explain concept of Balance Scorecard. Ans: The Balanced Scorecard (BSC) is a performance management tool which began as a concept for

Ans: Expense Centre: A centre or a unit of an organisation for whom a standard amount of cost to be incurred is predetermined and its performance its performance is evaluated by making a comparison between standard and actual costs is known as an expense centre. Any difference between standard and actual cost should be segregated further under two heads viz., controllable and un-controllable for an objective evaluation of expense centre performance.

There are two types of expense centre Engineered expense centre Discretionary expense centre

Engineered expense centre

Engineered expense centre are those where inputs or cost can be measured in advance with reasonable opportunity. They are usually formed in manufacturing operations where standard cost is used i.e. warehouse, distribution, trucking, etc. Engineered expenses are those expenses which are arrived at with reasonable reliability. E.g. Material cost, labor cost.

Engineered expense centre

Performance Measure for the RC is std.cost: - Std Cost of doing actual activity = Std. cost of unit activity * Quantum of Actual activity One can establish relationship between I & O , hence performance measurement is relatively easy

Discretionary Expenses Center

It includes administration and support units like financial, accounting , legal, Public Relation, HRD, R&D, advertising & Marketing. Discretionary expenses are those expenses which can not be established with perfect accuracy ensuring that managers adhere to the budgeted level of expenditures while successfully accomplishing the tasks of their center. They are those expenses which incurred as per organization policies derived from strategic planning & its goal and objectives.

-e.g. R&D, Advt. Dept, a Movie Project

Page 7: mesmerizers.files.wordpress.com  · Web viewQ. Explain concept of Balance Scorecard. Ans: The Balanced Scorecard (BSC) is a performance management tool which began as a concept for

Discretionary Expenses Center • Difficult to estimate Input (hence called MANAGED costs)• Output cannot be measured in monetary terms.• Difficult to establish optimal relationship between I and O• Performance Measure for the RC is Budgeted Input and Actual Input.

Q. Identify some Internal controlsAns: Internal controls - Internal control is a process-effected by an entity’s board of directors, management, and other personnel-designed to provide reasonable assurance regarding the achievement of objectives in the following categories:

Effectiveness & efficiency of operations Reliability of financial reporting Compliance with applicable laws & regulations

Internal controls can be detective, corrective, or preventive by nature. 1. Detective controls:They are designed to detect errors or irregularities that may have occurred. 2. Corrective controls -They are designed to correct errors or irregularities that have been detected. 3. Preventive controls-They are designed to keep errors or irregularities from occurring in the first place.

Q7. ‘Transfer pricing is an accounting tool’, comment.Ans: Transfer Pricing - When divisions transfer products or render services to each other, a transfer pricing is used to charge for the products or services.

Benefits of Transfer Pricing1. Divisions can be evaluated as profit or investment centers.2. Divisions are forced to control costs and operate competitively.3. If divisions are permitted to buy component parts wherever they can find the best price (either internally or externally), transfer pricing will allow a company to maximize its profits.

• Concept :-– Transfer price is defined as the value placed on transfer of goods or services among two or

more profit centers.– For selling profit center, the transfer price is major determinant of its revenue and hence its

profits.

Page 8: mesmerizers.files.wordpress.com  · Web viewQ. Explain concept of Balance Scorecard. Ans: The Balanced Scorecard (BSC) is a performance management tool which began as a concept for

– For buying profit center, the transfer price is major determinant of the expenses incurred and hence its profit.

– The price of inter divisional sales affects the selling divisional sales and buying divisional cost.

– Transfer price is fundamentally an attempt to simulate external market condition within the organization.

– Two divisions can be made completely independent of each other.• Objectives :

– It should provide each segment with the relevant information required to determine the optimum trade – off between company cost and revenue.

– It should induce goal congruent decisions. ( Decisions regarding division and company )– It should help measure the economic performance of individual profit centers.– The system should be easy to administer.

• Mechanism of Transfer Pricing :– Transfer price, means the value placed on a transfer of goods or services in transaction.– The FUNDAMENTAL PRINCIPLE is that the transfer price should be similar to the price that

would be charged if the product were sold to out side customers or purchased from out side supplier.

– When profit center of an organization buy product from and sell to one other, two decisions are to be carried out and reviewed periodically.

• Sourcing Decision: Should the company produce the product inside the company or purchase it from an out side vendor?

• Transfer Price Decision: If produced inside, at what price should be the product transferred to next centre?

– It starts from simple to extremely complex depending upon the nature of business.Hence transfer pricing is not actually an accounting tool but a behavioral tool to assist managers in management control.

Q. State the conditions under which transfer price mechanism is likely to induce Goal Congruence.The conditions under which transfer price mechanism is likely to induce Goal Congruence are as follows: Transfer price will induce goal congruence if all the conditions listed below exist.

– Competent People: Managers interested in long run and short run performance and staff involved in negotiation and arbitration of transfer price.

– Good Atmosphere: They should perceive that it is a mechanism.– Market Price: It should base on well established market price, which reflects same

conditions like quantity, quality, delivery time, etc.– Freedom to Source: Buying manager should have freedom to buy from out side and

selling manager should have freedom to sell out side.– Full of Information: Managers must have all information about the alternatives and cost.– Negotiation: Smooth mechanism for contract between business units.

Page 9: mesmerizers.files.wordpress.com  · Web viewQ. Explain concept of Balance Scorecard. Ans: The Balanced Scorecard (BSC) is a performance management tool which began as a concept for

• The Constraints on Sourcing :In actual all these conditions are not present the major short falls are:– Limited Market: Market for buying or selling is limited due to several reasons.

• Existences of internal capacity limit the development of external sales.• If company is sole producer of a differentiated product no out side source exists.• If company has developed significant facilities, it does not allow using out side

sources unless out side selling price approaches the company’s variable cost.– Excess or Shortage of Capacity :

• If selling unit can not sell all it can produce is excess capacity. The profit can not be optimizing if buying unit purchase from out side suppliers.

• If buying unit can not obtain product it requires from out side while selling unit is selling it out side is shortage of capacity. Out put of buying unit constrained.

Q. What are the objectives of Transfer Pricing? What is ideal transfer price in the situations of: - (a) Limited Market (b) Shortage of capacity in the industry. When do you use Cost Based Transfer Prices?ANS) Objectives of Transfer Pricing:

1) It should provide each business unit with the relevant information it needs to be determined the optimum trade-off between company costs & revenues.

2) It should induce Goal Congruent decisions--- that are; the system should be designed so that decisions that improve business unit profits will also improve company profits.

3) It should help measure the economic performance of the individual business units.4) The system should be simple to understand & easy to administer.

Ideal transfer price in the situations of: - (a) Limited Market:In the case of limited markets, the transfer price that best satisfies the requirements of a profit center system is the competitive price. Competitive prices measure the contribution of each profit center to the total company profits.Ideal transfer price in the situations of: - (b) Shortage of capacity in the IndustryIn the case of the Shortage of Capacity in the Industry, the transfer price would be the competitive price, and the other option is to develop Cost-based transfer prices.When to use Cost Based Transfer Prices:Cost Based Transfer Prices is used, if competitive prices are not available, transfer prices may be set on the basis of cost plus profit, even though such transfer prices may be complex to calculate & the results less satisfactory than a market-based price. Two decisions must be made in a cost-based transfer price system:

1. How to define cost &2. How to calculate the profit markup.

Q. “Internal Audit is one of the techniques of control available to the top management for ensuring continuous improvement for better performance”. Comment upon this statement analyzing it highlighting benefits accruing to the top management.Ans:- Internal audit can be viewed from two different perspectives-the traditional perspective and the modern perspective. Viewed from a Traditional perspective, internal audit is found to play the following roles:

Page 10: mesmerizers.files.wordpress.com  · Web viewQ. Explain concept of Balance Scorecard. Ans: The Balanced Scorecard (BSC) is a performance management tool which began as a concept for

Check whether the existing controls are effective and adequate. Check whether the financial reports and other records show the actual results of the company. Check whether the sub-units of the organization are following the policies and procedures laid down

by the management.

The Traditional concept of internal auditing has a narrow scope whereas the Modern concept has wider scope. The fact that the modern internal auditing is wider is reflected in the new definition of internal auditing given by the Institute of Internal Auditors,“An independent appraisal function established within an organization to examine and evaluate its activities as a service to the organization. The objective of internal auditing is to assist members of the organization in the effective discharge of their responsibilities. To this end, internal audit furnishes them with analyses, appraisals, recommendations, counsel and information concerning the activities reviewed.”

This definition implies that an internal auditor has to go beyond checking the books of account and related records. He has to appraise the various operational functions of an organization and provide recommendations about these.

Thus, according to the modern concept of internal auditing, the internal auditor is involved in conduction a review of operations, and internal audit and operational audit are almost synonymous.Need for Internal Auditing:-The need for internal audit is determined by the increasing size and complexity of organizational operations. Many organizations operate in number of countries and therefore have a large number of employees. In order to avoid discrepancies from creeping into their systems, processes, and operations, such organization appoint a team of specialists called internal auditors to monitor, track and report such discrepancies, inefficiencies of personnel in the concerned departments.

Benefits of Internal audit to Top Management:-Internal audit can be beneficial to most organizations as well as top management because, if planned properly, it provides management with a methodology to identify those risks that may prevent the organization from meeting its objectives.

For example, if a company has a strategic objective to raise Rs.20 million in loans to build a new facility there are a number of risks that may prevent that from occurring. One risk may be the external factor of increased interest rates. Another risk may be internal risk that management does not qualify for credit because of covenants they will not be able to meet. Financial costs of internal audit will vary based upon the size and goal of the internal audit function. Additionally, the cost will be based upon the resources used to perform the work (outsource, co-source, in-house). The most significant non-financial cost may be a negative reputation of the internal audit role throughout the organization. If the function is not properly established, socialized and executed then the validity of the function could be jeopardized.

Page 11: mesmerizers.files.wordpress.com  · Web viewQ. Explain concept of Balance Scorecard. Ans: The Balanced Scorecard (BSC) is a performance management tool which began as a concept for

Internal audit can be a value-add activity but often times it is strictly a policing function, which is sadly an example when the cost of internal audit usually does not exceed its benefit. Key Benefits of Internal Audit:- Inform the management on their possibility of achievement of organizational objectives. Better focus on high risk areas. Strengthens the planning process. As a means to help managements identify opportunities and downsize threats.

Q. ‘Internal Auditing means nothing but policing’, comment.Ans : Internal auditing is a profession and activity involved in helping organisations achieve their stated objectives. It does this by utilizing a systematic methodology for analyzing business processes, procedures and activities with the goal of highlighting organizational problems and recommending solutions. Professionals called internal auditors are employed by organizations to perform the internal auditing activity.

The scope of internal auditing within an organization is broad and may involve topics such as the efficacy of operations, the reliability of financial reporting, deterring and investigating fraud, safeguarding assets, and compliance with laws and regulations.Internal auditing frequently involves measuring compliance with the entity's policies and procedures. However, Internal auditors are not responsible for the execution of company activities; they advise management and the Board of Directors (or similar oversight body) regarding how to better execute their responsibilities. As a result of their broad scope of involvement, internal auditors may have a variety of higher educational and professional backgrounds.

Publicly-traded corporations typically have an internal auditing department, led by a Chief Audit Executive ("CAE") who generally reports to the Audit Committee of the Board of Directors, with administrative reporting to the Chief Executive Officer.

Role in internal controlInternal auditing activity is primarily directed at improving internal control. Internal control is broadly defined as a process, effected by an entity's board of directors, management, and other personnel, designed to provide reasonable assurance regarding the achievement of objectives in the following internal control categories:

Effectiveness and efficiency of operations. Reliability of financial reporting. Compliance with laws and regulations.

Management is responsible for internal control. Managers establish policies and processes to help the organization achieve specific objectives in each of these categories. Internal auditors perform audits to evaluate whether the policies and processes are designed and operating effectively and provide recommendations for improvement.

Page 12: mesmerizers.files.wordpress.com  · Web viewQ. Explain concept of Balance Scorecard. Ans: The Balanced Scorecard (BSC) is a performance management tool which began as a concept for

Hence we can say that internal auditing means nothing but policing.

MANAGEMENT CONTROL PROCESS:The management control process for ongoing operating activities has the following four phases: (I) Programming (2) Budgeting (3) Execution (4) Evaluation.

Control process in case of non-operating activities such as a project consists of the above phases except the two phases programming and budgeting, are combined into a single activity, project planning, There are differences in the nature of a project and that of operating activities. A project generally has a single objective and ongoing operating activities have multiple objectives. A project comes to an end when the objective is accomplished. An ongoing operating organisation intends to operate indefinitely. In some cases, the completion of a project may result into an ongoing operating organisation although this may involve complex management control problems.The discussion of the four phases of management control - Programming, Budget Preparation, Execution, Evaluation – are as follows:(1) Programming: Programming is defined as making programmes by top/senior management in terms of organizational goals and strategies and deciding the funds and resources needed to accomplish the programmes. Programmes can be made about developments of new products, research and development activities, merger, takeover, other activities.

Programming is a long range plan, covering period of approximately 5 future years. The reason is that if programming is made for a shorter period, the results and benefits of programming cannot be realised within this period. Some organisations like public utilities prepare long-range plans for even a period of twenty years. Because of the relatively long-time plan, only rough estimates are possible for revenues, expenses and capital expenditure.

The task of programming is done by senior/top management and the managers of divisions or principal responsibility centers, assisted by their staffs. The managers of lower responsibility centres generally do not participate in the programming process. Although the staff of toe divisions making the programming can participate, the decisions on the different aspects of the plan are made by the line managers. Programming is time-consuming and expensive. The most significant expense is the time devoted to it by management, but it also involves a special programming staff and considerable paperwork. A formal programming process is not worthwhile in some organisations. It is desirable in organisations that have the following characteristics. (1) Its top management is convinced that programming is important. Otherwise programming is likely to be, or to become, a staff exercise that has little impact on actual decision-making.

Page 13: mesmerizers.files.wordpress.com  · Web viewQ. Explain concept of Balance Scorecard. Ans: The Balanced Scorecard (BSC) is a performance management tool which began as a concept for

(2) It is relatively large and complex. In small, simple organisations, an informal understanding of the organisation’s future directions is adequate for making decisions about resource allocations, which is a principal purpose of preparing programme. (3) Considerable uncertainty about the future exists, but the organisation has the flexibility to adjust to changed circumstances. In a relatively stable organisation, a programme may be unnecessary; the future is sufficiently like the past so that the programme would be only an exercise in extrapolation. If the future is so uncertain that reasonably reliable estimates cannot he made, preparation of a formal programme is a waste of time.

In summary, a formal programming process is not needed in small, relatively stable organisations, and it is not worthwhile in organisations that cannot make reliable estimates about the future or in organisations whose top management does not prefer to manage in this fashion.

(2) Budgeting: Budgeting is formal financial plan for each year. A budget, known as short-range plans, is a technique of expressing revenues, physical targets like production and sales, profit, assets and liabilities usually for a period of one future year.

Budget has the functions of motivating managers, coordinating activities, communicating to person within an organisation, providing standards for judging actual performances and acting as a control tool.

Programming and budgeting differ from each other in following respects: (1) Programming involves senior managers: operating managers ordinarily are not involved. Budgeting involves operating managers as well as senior managers.

(2) Staff personnel have a considerable input to the programming process but relatively less input to the budgeting process. In a large organization the programming staff is distinct from the budget staff. The programming staff (perhaps called planning staff or analysis staff) is skilled in analysing proposed new programmes; the budget stall is skilled in finding soft spots in proposed budgets.

(3) The programme structure consists of programme and major project; it includes both capital expenditure and operating items and it covers a period of several years. The budget is structured by responsibility center (which may or may not cut across programs) the focus is on operating revenues and expenses and it typically is for a single year.

(4) Budget preparation is done under greater time pressure and is more hectic than programming.

(5) A programme is a broad brush sketch of the future. A Budget has more detail, both because it is a fairly specific guide to operating decisions and also because it will be s subsequently used to evaluate the performance of individual managers. Also, “management by objectives” are incorporated in the budget, but not usually in the programme.

Page 14: mesmerizers.files.wordpress.com  · Web viewQ. Explain concept of Balance Scorecard. Ans: The Balanced Scorecard (BSC) is a performance management tool which began as a concept for

(6) Programming decisions can have consequences of great magnitude. Budgeting decisions are typically much less significant, because they are made within the context of the current level of operating activities, except as those activities will be affected by programme decisions.

(7) Behavioral considerations are in the programming process. The approved budget is a bilateral commitment; the programme s not a commitment. Because the budget will be used to evaluate performance, operating managers tend to be much more concerned with the numbers in the budget than with the numbers in the programme.

(3) Executing: After the budget preparation budgeting is used as a tool for coordinating the actions/individuals and department within the organisatlon. In fact, within the execution phase, task control is done to ensure that action and performances matched with the planned or desired results. While performing, the manager’s goal is to achieve budgeted targets. However, compliance to budget is not necessary if the plans given in the budget are found as not the best way of achieving the objectives. Adherence to budget is not necessarily good, and departure from it is not necessarily bad.

After execution, actual performances and results are compared with the budgeted plans and the target and variances report are prepared which highlight the variances between the two and the causes for such variances. Variance reports should separate controllable items from non-controllable items, determine the effect of changes in volume on revenues and costs and if possible, should mention changes in other circumstances affecting the variances. Variance reports lead to corrective actions subsequently. Variance reports should be prepared timely and promptly and contain all useful information for helping the managers take corrective actions.

(4) Evaluation: Management control process ends with the evaluation phase in which the performance of managers are evaluated. Since, it is an after-event exercise, the evaluation does not affect what has happened. However evaluation phase acts like a powerful stimulus as employees know that their performances will be subsequently evaluated. Also, on the basis of performance evaluation, the future budgets and plans are revised.

Expense Centers: Expense centers are responsibility centers whose inputs are measured in monetary terms, but whose outputs are not. There are two types of Expense centers: Engineered and Discretionary. Engineered Expense centers: have the following characteristics:

(i) Their input can be measured in monetary terms.(ii) Their output can be measured in physical terms. (iii) The optimum rupee value of input required to produce one unit of output can be determined.

Engineered expense canters are usually found in manufacturing operations. They could he engineered expense centers in marketing like warehousing, distribution, trucking; in administration and support departments also.

Page 15: mesmerizers.files.wordpress.com  · Web viewQ. Explain concept of Balance Scorecard. Ans: The Balanced Scorecard (BSC) is a performance management tool which began as a concept for

In engineered expense center, output multiplied by the standard cost of each unit produced measures what the finished product should have costs. The difference between the theoretical cost and actual cost represent the efficiency of the expense center. Besides this cost the managers of responsibility center is also responsible for quality and timely delivery of product and training and development of his employees.

Discretionary Expense centers: Discretionary expense centers include administrative and support units, research and development, and most marketing activities. The output of these centers cannot be measured in monetary terms. The term discretionary means management exercise its judgment about what the costs should be taking into account its strategy and competitive environment. In a discretionary expense center the difference between budget and actual expense is not a measure of efficiency. It only means that the manager has lived within its budget, which may not necessarily indicate efficient performance.

General control consideration:Budget Preparation: Management formulates budget for a discretionary expense center by determining the magnitude of the job that needs to be done. The work done by a discretionary expense center tails into two categories: Continuing and Special.

Continuing Work is done consistently from year to year such as preparation of financial statement, by controller’s department: Special work is a one slot project. E.g. Developing and installing a profit budgeting system in newly acquired division. MBO is used for budget preparation in a discretionary cost center. ‘Where in a budgetee proposes to do specific jobs and suggest measures to be used in performance evaluation.’

The planning function for discretionary expense center is usually carried out in two ways: (a) Incremental budgeting (b) Zero-based budgeting

Incremental Budgeting:In this method current level of expenses is taken as a starting point. This amount is adjusted for inflation, anticipated changes in workload of continuing jobs, special job and if data are readily available, the cost of comparable jobs in similar units. It has 2 drawbacks:Current level of expenditure is accepted and not reexamined during the process of budget preparation. Manager of these centers wants to increase the current level of services and thus tend to request additional resource which if they make sufficiently strong case- are usually provided. This tendency is called Parkinson’s Second Law.

Zero Based Review:

Page 16: mesmerizers.files.wordpress.com  · Web viewQ. Explain concept of Balance Scorecard. Ans: The Balanced Scorecard (BSC) is a performance management tool which began as a concept for

A thorough review is made of each expense center. This review attempts to ascertain de novo, i.e. from scratch, the resources actually required to carry out each activity. This analysis estab1ishes a new base, the annual budget simply tries to keep the cash in line with this new base. Certain basic questions raised in this analysis are:Should the function under review be performed at all? Does it add value from the stand point of end use customer? What should the quality level be? Are we doing too much? Should the function be performed in this way? How much should it cost? We try to bench mark our expense with other similar organizations? Zero base review is time consuming. It is traumatic for the manager whose performance is being reviewed.

These days lot of companies resort to restructuring, right sizing, business process re-engineering to improve the profitability.

Measurement and Controlling Assets employed:When the profit earned by a business unit is compared with the assets employed in earning it, the business unit is termed as Investment center. There are two methods of relating profit to the investment base (I) ROI Return on 1nvestment (2) EVA - Economic Value Added. Structure of Analysis: The objectives of the Investment center are: 1) To provide information that is useful in making sound decisions about assets employed and to motivate managers to make these sound decisions that are in the best interest of the organization. 2) To measuring the performance of the business unit as economic entity.

Focusing on profits, without considering the assets employed to generate those profit is an adequate basis for control. Except in certain service types of organization in which the amount of capital is significant.Unless the amount of assets employed is taken into account, it is difficult for Senior Management to compare the profit performance of one business unit with that of the other units or to similar outside companies.

In general business unit managers have two performance objectives. First they should generate adequate profits from the resources at their disposal. Second, they should invest in additional resources only when the investment would produce adequate return. Conversely, they should dis—invest if the existing assets do not earn adequate return.

Return on investment (ROI is a ratio. The numerator is income, as reported on the income statement. The denominator is assets employed.

Economic value added is a Rupee amount rather than a ratio. It is found by subtracting capital charge from the net operating profit. EVA is conceptually superior to ROI, though ROI is still widely used.

Page 17: mesmerizers.files.wordpress.com  · Web viewQ. Explain concept of Balance Scorecard. Ans: The Balanced Scorecard (BSC) is a performance management tool which began as a concept for

Measuring Assets Employed:In deciding what- investment base to use to evaluate investment center manager, headquarter asks two questions:

(i) What practices will motivate the business unit managers to use their assets most efficiently and to acquire the proper amount, and kind of new assets?

(ii) What practices best measures the performance of the unit as an economic entity?

Cash: Most companies control cash centrally because this permits use of a smaller cash balance it needed to weather the unevenness of its cash inu1ovs and outflows. Business unit cash balances are only the ‘‘float’ between daily receipts and daily disbursements. For the purpose of comparison with outside company, the units need to slow higher cash balance. GM reports 4.5% of annual sales as cash.

Receivables: Business unit managers can influence the level of’ receivable indirectly, by their ability to generate sales, and directly by establishing credit terms and approving individual credit accounts and -credit limits, and by their vigour in collecting overdue accounts. Receivables are often included as Sundry Debtors less Provision for bad and doubtful debt.

Inventories are often recorded at end of period amounts even though intra period average would conceptually better. Inventories should be valued at standard or average cost and these costs should be used to measure the cost of sales in Profit and Loss Account. If work in process inventory is finance, by advance payments from the customer, this should be subtracted.

Working Capital in General: Some companies use Gross working capital as investment base. This method is sound if the business unit manager can not influence accounts payable and other current liabilities. Other company use Net Working Capital for Investment base. This method provides a good measure of capital provided by the corporation.

Property, Plant and Equipment: In financial accounting, fixed asset are initially recorded at their acquisition cost, and this cost is written of over the asset’s useful life through depreciation. Most companies use a similar approach in measuring profitability of the business unit’s asset base. This causes some serious problems in using the system fur its intended purposes.

Investment Centre:An investment centre is responsible for both profits and investments. The investment centre manager has control over revenues, expenses, and the amounts invested in the centre’s assets. He also formulates the credit policy which has a direct influence on debt collection, and the inventory policy which determines the investment in inventory. The manager of an investment centre has more authority and responsibility than the manager of either a cost centre or a profit centre. Besides controlling costs and revenues, he has investment responsibility too. Investment of asset responsibility means the authority to buy, sell, and use divisional assets.

Page 18: mesmerizers.files.wordpress.com  · Web viewQ. Explain concept of Balance Scorecard. Ans: The Balanced Scorecard (BSC) is a performance management tool which began as a concept for

Q. What are the objectives of Transfer prices? Under which conditions a Transfer pricing mechanism is likely to induce Goal congruence?

ANS: Today’s organizational thinking is oriented towards decentralization. One of the principal challenges in operating a decentralized system is to device a satisfactory method off accounting for the transfer of goods and services from one profit center to another in companies that have a significant number of these transactions.

OBJECTIVES OF TRANSFER PRICING: If two or more profit centers are jointly responsible for product development, manufacturing and

marketing, each should share in the revenue generated when the product is finally sold. The transfer price is the mechanism for distributing this revenue. The transfer price should be designed so that it accomplishes the following objectives:It should provide each business unit with the relevant information it needs to determine the optimum trade-off between company costs and revenues.It should induce goal congruent decisions – that is, the system should be designed so that decisions that improve business unit profits will also improve company profits.It should help measure the economic performance of the individual business unitsThe system should be simple to understand and easy to administer.

A market price- based transfer price will induce goal congruence if all of the following condition exists. Rarely, if ever, will all these conditions exist in practice. The list, therefore, does not set forth criteria that must be met to have a transfer price. Rather,, it suggests a way of looking at the situation to see what changes should be made to improve the operation of the transfer price mechanism.

COMPETENT PEOPLE:Ideally, managers should be interested in the long-run as well as the short-run performances of their responsibility centers. Staff people involved in negotiation and arbitration of transfer prices also must be competent.

GOOD ATMOSPHEREManagers must regard profitability, as measured in their income statements, as an important goal and a significant consideration in the judgment of their performance. They should perceive that the transfer prices are just.

A MARKET PRICEThe ideal transfer price is based on a well-established, normal market price for the identical product being transferred- that is, a market price reflecting the same conditions (quantity, delivery time, and quality) as the product to which the transfer price applies. The market price may be adjusted downward to reflect savings accruing to the selling unit from dealing inside the company. E.g. there would be no bad debt expense, and advertising and selling costs would be smaller when products are transferred from one

Page 19: mesmerizers.files.wordpress.com  · Web viewQ. Explain concept of Balance Scorecard. Ans: The Balanced Scorecard (BSC) is a performance management tool which began as a concept for

business unit to another within the company. Although less that ideal, a market price for a similar, but not identical, product is better than no market price at all.

FREEDOM TO SOURCEAlternatives for sourcing should exist, and managers should be permitted to choose the alternative that is in their own best interests. The buying manager should be free to buy from the outside, and the selling manager should be free to sell outside. In these circumstances, the transfer price policy simply gives the manager of each profit center the right to deal with either insiders or outsiders at his or her discretion. The market thus establishes the transfer price. The decision as to whether to deal inside or outside also is made by the marketplace. If buyers cannot get a satisfactory price from the inside source, they are free to buy from the outside.

This method is optimum if the selling profit center can sell all of its products to either insiders or outsiders and if the buying center can obtain all of its requirements from either outsiders or insiders. The market price represents the opportunity costs to the seller of selling the product inside. This is so because if the product were not sold inside, it would be sold outside. From a company point of view, therefore, the relevant cost of the product is the market price because that is the amount of cash that has been forgone by selling inside; the transfer price represents the opportunity cost to the company.

FULL INFORMATION:Managers must know about the available alternatives and the relevant costs and revenues of each.

NEGOTIATION:There must be a smoothly working mechanism for negotiating “contract” between business units.If all of these conditions are present, a transfer price system based on market price would induce goal congruent decisions, with no need for central administration.

(B) Explain advantages and disadvantages of two step pricing and profit sharing methods.ANS: Transfer pricing can create a significant problem in integrated companies. The profit center that finally sells to the outside customer may not even be aware if the amount of upstream fixed costs and profit included in its internal purchase price.

To handle this problem is to establish a transfer price that includes two charges. First, for each unit sold, a charge is made that is equal to that standard variable cost of production.Second,a periodic (usually monthly) change is made that is equal to the fixed costs associated with the facilities reserved for the buying unit.

Q. Write short notes:A) Strategy formulation and task controlStrategy formulation is the process of deciding on the goals of the organization and the strategies for attaining these goals. Goals are timeless, they exist until they are changed, and they are changed only

Page 20: mesmerizers.files.wordpress.com  · Web viewQ. Explain concept of Balance Scorecard. Ans: The Balanced Scorecard (BSC) is a performance management tool which began as a concept for

rarely. For many businesses, earning a satisfactory return on investment is an important goal for others attaining a large market share is equally important. Non-profit organizations also have goals; they seek to provide the maximum services possible with available funding.

Strategies are big plans, important plans. They state in a general way the direction in which senior management wants the organization to move. The need for formulating strategies usually arises in response to a perceived threat (a shift in consumer taste, new government regulations) or opportunity (technological innovations).strategies to address a threat or opportunity can arise from anywhere in an organization and at any time. Complete responsibility for strategy formulation should never be assigned to a particular person or organizational unit. Virtually anyone might come up with a “bright idea”, which, after analysis and discussion, can form the basis for a new strategy.

Strategy formulation is essentially unsystematic. Threats, opportunities, and new ideas do not occur at regular intervals, thus, strategic decisions may be made at any time. Furthermore, the analysis of a proposed strategy varies with the nature of the strategy. Strategy analysis involves much judgment, and the numbers used in the process are usually rough estimates.Analysis of a proposed strategy usually involves relatively few people- the sponsor of the idea, headquarters staff, and senior management.

TASK CONTROL:Task control is the process of ensuring that specified tasks are carried out effectively and efficiently. It is transaction oriented-that is, it involves the performance of individual tasks according to rules established in the management control process. Task control often consists of seeing that these rules are followed, a function that in some cases does not even require the presence of human beings. Many task control activities are scientific; that is, the optimal decision or the appropriate action of bringing an out-of-control condition back to the desired state is predictable within acceptable limits. For instance, the rules for economic order quantity determine the amount and timing of purchase orders. Task control is the focus of many management science and operations research techniques.

Most of the information in an organization is task control information. Many of an organization’s central activities- including procurement, scheduling, order entry, logistics, quality control and cash management – are task control systems. Some of them, though mechanical, can be extremely complicated.

Certain activities that were once performed by managers are now automated and have thus become task control activities. This shift from management control to task control frees some of the manager’s time for other management activities.

Q. Discuss and illustrate difference and similarities between management control, strategy formulation and task control.Management Control

Page 21: mesmerizers.files.wordpress.com  · Web viewQ. Explain concept of Balance Scorecard. Ans: The Balanced Scorecard (BSC) is a performance management tool which began as a concept for

Management control is the process by which managers influence other members of the organization to implement the organization's strategies. Several aspects of this process are amplified below. Management Control Activities Management control involves a variety of activities, including:

· Planning what the organization should do.· Coordinating the activities of several parts of the organization.· Communicating information.· Evaluating information.· Deciding what, if any, action should be taken.· Influencing people to change their behavior.Management control does not necessarily require that all actions correspond to a previously determined plan, such as a budget. Such plans are based on circumstances believed to exist at the time they were formulated. If these circumstances have changed at the time of implementation, the actions dictated by the plan may no longer be appropriate. While a thermostat responds to the actual temperature in a room, management control involves anticipating future conditions in order to ensure that the organization's objectives are attained. If a manager discovers a better approach-one more likely than the predetermined plan to achieve the organization's goals-the management control system should not obstruct its implementation. In other words, conforming to a budget is not necessarily good, and departure from a budget is not necessarily bad.Strategy FormulationStrategy formulation is the process of deciding on the goals of the organization and the strategies for attaining these goals. In this book, we use the word goals to describe the broad overall aims of an organization, and the term objectives to describe specific steps to accomplish the goals within a given time frame.

Goals are timeless; they exist until they are changed, and they are changed only rarely. For many businesses, earning a satisfactory return on investment is an important goal; for others, attaining a large market share is equ~ important. Nonprofit organizations also have goals; in general, they seek to provide the maximum services possible with available funding. In the strategy formulation process, the goals of the organization are usually taken as a given, although on occasion strategic thinking can focus on the goals themselves.Task ControlTask control is the process of assuring that specified tasks are carried out effectively and efficiently. Task control is traitsaction-oriented-that is, it involves the performance of individual tasks according to rules established in the management control process. Task con to 1 often consists of seeing that these rules are followed, a function that in some cases does not even require the presence of human beings. Numerically controlled machine tools, process control computers, and robots are mechanical task control devices. Their function involves humans only when the latter prove less expensive or more reliable; this is likely to happen only if unusual events occur so frequently that programming a computer with rules for dealing with these events is n'ot worthwhile.Many task control activities are scientific; that is, the optimal decision or the appropriate action for bringing an out-of-control condition back to the desired state is predictable within acceptable limits. For instance, the rules for economic order quantity determine the amount and timing of purchase orders.Task control is the focus of manv management science and operations research techniques. Most of the information in an organization is task control information.

Q. Describe and illustrate significance of human behavior patterns in management control.ANS: Management control systems influence human behavior. Good management control systems influence behavior in a goal congruent manner; that is, they ensure that individual actions taken to achieve personal goals also help to achieve the organization's goals.We begin this chapter by explaining the concept of goal congruence, describing how it is affected both by informal actions and by formal systems. The formal systems can be divided into two categories: "rules," broadly defined, and systematic methods for planning and for maintaining control.Different structures are used to implement strategies in various types of organizations; an effective management control system should be designed to fit the particular structure. In the final section of the

Page 22: mesmerizers.files.wordpress.com  · Web viewQ. Explain concept of Balance Scorecard. Ans: The Balanced Scorecard (BSC) is a performance management tool which began as a concept for

chapter, we describe the role of the controller-the person responsible for the design and operation of the management control system. Senior management wants the organization to attain the organization's goals. But the individual members of the organization have their own personal goals, and they are not necessarily consistent with those of the organization. The central purpose of a management control system, then, is to ens1.fre (insofar as is feasible) a high level of what is called "goal congruence." In a goal congr:uent process, the actions people are led to take in accordance with their perceived ;elfinterest are also in the best interest of the organization.Obviously, in our imperfect world, perfect congruence between individual goals and organizational goals does not exist-if for no other reason than that individual participants usually want as much compensation as they can get while the organization maintains that salaries can go only so high without adversely affecting profits. .

Q. Explain possible reasons for failure of Balanced Score Card? Ans: The BSC is a system of financial and nonfinancial measures that reflect a balance between leading and lagging indicators of performance and between outcome measures and measures that drive performance. These measures are selected to complement the organization's strategy.Moreover, many BSC implementations fail. Paul McCunn, a KPMG management consultant, estimates the overall failure rate at 70%.3 Those survey participants whose BSC implementation has failed may be reflected in our survey as nonusers. Breakdowns in communication and difficulty in translating the strategy into action are common reasons for failure.4 It is often difficult for employees to know what to do to improve performance. Measures and targets are often chosen by management and conveyed to the employees. Getting employees involved in picking measures and setting targets can help them to be more committed to reaching the goals.PROVIDING INCENTIVE COMPENSATION: EASIER SAID THAN DONESixty percent of regular users of the BSC provided financial incentives to employees for meeting or exceeding targets that were congruent with BSC measures, according to our survey. At the top level, CEOs are given stock options to provide an incentive to increase share prices. Business unit managers, middle managers, and front-line supervisors can have their bonuses and salary increases linked to their meeting or exceeding targets.Linking BSC measures to compensation is difficult, though, and carries some risk. The difficulty comes in determining the relative weights of the various performance measures on the scorecard.There are two methods for determining the weightings of multiple performance measures for an incentive pay scheme. One method uses predetermined weights for each measure in a prescribed formula. For example, 15% of the incentive pay may be based on operating income, 10% on revenue growth, 10% on customer satisfaction, 5% on internal defect rates, etc. The percentages of weights would be determined before the beginning of the period and could not be changed by the supervising manager.Formula-based systems usually ignore most of the BSC measures, however, because of the complexity of basing bonuses on a formula with 20 or more variables. Incentive pay based on a few variables will focus the manager's attention on those variables that can increase his or her compensation. The remaining variables will tend to be ignored even though they are also linked to the organization's strategy. An incentive pay plan that focuses on only a few variables will also tend to focus on financial measures, which are focused on short-term results and can be manipulated easily.Another method allows the supervising manager to subjectively determine the measurement weights at the end of the period. Financial and BSC results are reported for a typical accounting period, such as a

Page 23: mesmerizers.files.wordpress.com  · Web viewQ. Explain concept of Balance Scorecard. Ans: The Balanced Scorecard (BSC) is a performance management tool which began as a concept for

month or quarter, and targets are established for each period. Kaplan and Norton assert that BSC implementation makes it easier for managers to set bonuses and other incentive rewards subjectively.5 They believe that the development of the performance measures with their targets allows supervising managers an opportunity to better observe the performance of subordinate managers and assess their abilities.

The supervising managers then can set incentive rewards (such as bonuses and pay raises) subjectively based on their overall assessment of the scorecard performance. Kaplan and Norton believe that a subjective weighting scheme allows supervising managers to utilize key measures for a defined time period and ignore other BSC measures that are not key for that period."OUT-GAMING" MANIPULATIVE MANAGERSThe subjective weighting scheme helps to ensure that subordinate managers cannot "game" the system. One way to game the pay system is to trade performance on one measure to meet or exceed the target on another measure. For example, an unscrupulous purchasing manager whose bonus is based primarily on material prices might order excessive quantities of raw materials in order to get quantity discounts. This practice could completely ignore the organization's emphasis on keeping inventory levels low.Another example would be a business unit manager whose bonus is heavily weighted toward revenue growth. That growth might be achieved by discounting prices, giving favorable payment terms, and selling to customers with poor credit ratings.These attempts to trick the system should be apparent to the supervising manager with a BSC. The supervising manager can thwart attempts to game the system by failing to pay incentives in such situations. In this instance, the supervising manager has subjectively determined the weights of the various measures in the incentive pay plan.There are some risks in subjectively determining measurement weights. The following three claims were made by employees of a major financial services firm that attempted to link the BSC to its incentive compensation system. The firm later abandoned the BSC in favor of a formulaic calculation of incentive pay based on revenues.* Too much emphasis may be placed on financial measures, which are lagging indicators of performance that focus too much attention on gaining short-term results at the expense of long-term performance. This happens because managers have historically used financial measures in incentive pay plans.* It may be difficult for the employee to understand just how bonuses were determined.* Those who receive less incentive compensation than they expected may allege favoritism or bias.Even with the problems of linking the BSC to management compensation noted above, it may be necessary to make this connection in order to implement this management tool successfully because managers usually focus on items that affect their compensation directly. In fact, the survey provides evidence of a correlation between improved operating performance and the linking of the BSC to compensation. In another survey result, we found that 65% of the respondents who did not see an increase in operating performance did not use the BSC to determine management compensation. Of those respondents who did see an increase in operating performance, 66% used the BSC to determine management compensation.

Page 24: mesmerizers.files.wordpress.com  · Web viewQ. Explain concept of Balance Scorecard. Ans: The Balanced Scorecard (BSC) is a performance management tool which began as a concept for

While linking BSC performance goals to compensation may present additional complexity, this step seems to be a valuable tool in implementing the BSC successfully.

Q. What are advantages in conducting internal Audit?Ans: The internal audit is an essential—and now much discussed—part of business. The Institute of Internal Auditors, an international association of internal auditors, offers this definition: “An independent, objective assurance and consulting activity designed to add value and improve an organization’s operations. It helps an organization accomplish its objectives by bringing a systematic, disciplined approach to evaluate and improve the effectiveness of risk management, control, and governance processes.” To this end, internal auditing furnishes staff with analyses, appraisals, and recommendations relating to these activities.An internal audit offers several advantages.

It directs management’s attention to the key business issues. The audit analyzes weaknesses in the system of control, and becomes the basis for practical recommendations for improvement.

It gives management confidence when controls are operating satisfactorily. It identifies opportunities for improving efficiency and effectiveness. It gives early notice of potential problems, so that management can take action to head them off. It dispenses the need to employ external consultants to act as internal auditors hence saving large

sum of money. This is even especially true when an internal audit department is properly run with well trained and experienced internal auditors;

The internal auditors are intimately acquainted with the business as they are continuously employed in the same concern and have access to much confidential information and to all levels of management. Hence, they really are “special” personnel who have very in depth inner knowledge which can then contribute to the company;

The Internal Audit maintains a group of highly skilled people available to cope with non-recurring and exceptional jobs which no many employee could deal with efficiently and effectively;

It ensures that the organization detailed standard policy and procedures are running smoothly. This compared to the external auditors’ primary role of the ability to express the true and fair view of the clients’ financial statements audit;

Internal auditors are invaluable in areas like operational audits, constant examination of internal check controls, the detailed application of normal auditing method and detailed review of the various type of management reporting;

Last but not least, it provides an excellent training ground for future executives. Trainee personnel obtain intimate knowledge of the business which they can study problems of all kinds at different levels

Q. Illustrate difference between Financial and Management audit?Ans:Points Financial Audit Management AuditPurpose The Purpose of financial audit is to

find out whether the accounts The Purpose of management audit is to review the management

Page 25: mesmerizers.files.wordpress.com  · Web viewQ. Explain concept of Balance Scorecard. Ans: The Balanced Scorecard (BSC) is a performance management tool which began as a concept for

present true and fair financial state of affairs of the concern.

operations with a view to bring about necessary improvements.

Nature The financial audit is a post-mortem examination.

The management audit is a preventive as well as curative check.

Basis In financial audit, only financial data from internal sources are used for the purpose of examination.

The management audit is based on financial as well as non-financial data derived from internal and external sources.

Statutory Provision

Financial audit is legally compulsory and legal rules and regulations have to be observed for that.

There is no statutory provision for undertaking the management audit, and it is not legally compulsory but discretionary.

Qualification In financial audit, it can be undertaken by the chartered accountants only.

There is no prescribed qualification for the management auditor and any expert on the management may be asked to undertake it.

Process The financial audit examines financial accounts, statements and schedules and generally accepted audit techniques are applied for the purpose.

The management auditor examines the managerial activities and operations and adopts operations management technique for the purpose.

`Norms for Evaluation

In financial audit, the principles, conventions and postulates are used as norms.

In case of management audit, the generally accepted management standards or norms are used for the purpose of evaluation and examination.

Precision In financial audit, absolute precision is quite necessary.

In management audit, only relative precision is necessary.

Area of Study In financial audit, it is necessary to examine the accounts of the entire organisation.

In management audit the study may be related to the entire organisation or part.

Periodicity In financial audit, it has to be undertaken every year.

While there is no fixed for management audit.

Reporting In case of financial audit, the report of the financial audit has to be submitted to the shareholders, who appoint the auditor.

While the management auditor is appointed by the management and hence, the report has to be submitted to the shareholders, who appoint the auditor.

Liability The financial auditor can be held criminally liable under the

While the management auditor

Page 26: mesmerizers.files.wordpress.com  · Web viewQ. Explain concept of Balance Scorecard. Ans: The Balanced Scorecard (BSC) is a performance management tool which began as a concept for

Companies Act, 1956. cannot be held criminally liable.

Q. Explain briefly the likely features of an IDEAL management control System?Ans: Management control has been defined as the process by which manager influence other members of their organization to implement their strategies and also assure that the resources are obtained and used effectively in achieving its objective. The process of management control is carried out within the framework outlined by strategic planning. It is intended to achieve the planned objective as effectively and efficiently as possible within given parameter.Following are the silent features of M.C.S:1. Process: Management control systems consist of a process and a structure. The process refers to the set of action that takes place, such as programming, budgeting, operating, performance measuring, Reporting, analysis and communicating information. The structure refers too the organization set up indicating lines of authority, delegation, Responsibility and accountability.2. Managers: Management control is the process for the use of managers. It involves interaction with managers as well as subordinates.3. Strategies: They include strategic planning process which includes goals, Objectives and policies.4. Assures: The management insure the organization carry out strategies effectively and efficiently.5. Effectively: it indicates the satisfaction expressed by its customers for doing the job in time and in the manner desired by the management.6. Efficiently: it is the skill of producing maximum output with the minimum input.Characteristic features of management control system: It forces on programmers and responsibility centres. A programmer means and includes a project, product, goods and services that an organization undertakes to achieve its goal. Responsibility centre is an unit or sub unit in the organization usually headed by an manager, who become responsible and accountable for the performance of that unit. For the purpose of control the manager uses two types of data viz. planned data and actual data. Planned data relate to estimates, Budgets, Standards and forecast details of future events or activities. The control process is the set of actions such as programming, Budgeting and monitoring, evaluating, analyzing and reporting. It is contentious and an ongoing process. It is the total system covering all aspects of the company operations. Management control assures that all parts are balanced with one another. Unless management obtains information about each of them, Coordination between all the activities is not possible. The system is normally coordinated and integrated so that the same set of information can be used by the other for some other purpose. It means that although the data collected for one purpose may differ from the data collected from the other purpose, this data should be reconciled with one other. MCS is the set of interlocking sub system generating a single system. This unit is usually designed with financial figures and not just physical units of uinput or output. Because monetary unit is the only common denominator which can be used to express heterogeneous input and resources. The system tends to be rhythmic and not one time or an as hoc activity. It follows a regular time table like weekly, Monthly etc. In the budget preparation, actual operations, reporting, and reviewing the

Page 27: mesmerizers.files.wordpress.com  · Web viewQ. Explain concept of Balance Scorecard. Ans: The Balanced Scorecard (BSC) is a performance management tool which began as a concept for

procedure to be followed at each step in the process the time table for the completion of each process, the formats to be used for reporting all are set in the policy and procedure manual. All reports are prepared and forwarded to the persons mentioned in the circulation list at regular interval. Line and staff managers are equally involved in the control process, but more significant decisions are made by the line managers, not by the staff managers. Staff people collect, summarizes and present information that are useful in the planning process and system design, but the line managers judgment are mostly incorporated in the approval plans, because they are the persons who will influence others and their performance will be measured. Line managers are therefore the focal point of M.C.SFinally the purpose of the Management Control System is to encourage managers to take actions that are in the best interest of the company.

Q. What is a responsibility center? What are the types of the responsibility centers with sketches of each?Ans. Responsibility center:

It is an organization unit for which a manager is made responsible. The center’s manager and supervisor establish specific and measurable goals for the responsibility

center. The goals should promote the long-term interest of the organization.

Manager of Responsibility Center: Establish goals Promote long-term interests Promote coordination of each responsibility center’s activities

Types of the responsibility centers:Responsibility centers are classified into four types:

1 Cost centers2 Revenue centers3 Profit centers4 Investment centers

Discretionary Expense Centers

Work

Optimal relationship can be established

Manufacturing function

Inputs Outputs

(Dollar)

Engineered Expense Centers

(Physical)

Examples

Optimal relationship cannot be established

Page 28: mesmerizers.files.wordpress.com  · Web viewQ. Explain concept of Balance Scorecard. Ans: The Balanced Scorecard (BSC) is a performance management tool which began as a concept for

Revenue Centers

Profit Centers

Investment Centers

Expense center: Expense centers are responsibility centers whose inputs are measured in monetary terms, but whose outputs are not. There are two general types of expense centers: engineered and discretionary. These

WorkResearch & development function

Inputs Outputs

Work Marketing function

Inputs Outputs

(Dollar only for costs directly incurred) (Dollar Revenue)

Inputs not related to outputs

WorkBusiness unit

Inputs Outputs

(Dollar Costs) (Dollar Profits)

Inputs are related to outputs

WorkBusiness unit

Inputs Outputs

(Dollar Costs)(Dollar Profits)

Profits are related to capital employed

(Dollar) (Physical)

Page 29: mesmerizers.files.wordpress.com  · Web viewQ. Explain concept of Balance Scorecard. Ans: The Balanced Scorecard (BSC) is a performance management tool which began as a concept for

labels relate to two types of cost. Engineered costs are those for which the “right” or “proper” amount can be estimated with reasonable reliability – for example, a factory’s costs for direct labor, direct material, components, supplies, and utilities. Discretionary costs (also called managed costs) are those for which no such engineered estimate is feasible. In discretionary expense centers, the cost incurred depend on management’s judgment as to the appropriate amount under the circumstances. Revenue center: It is a responsibility center whose members control revenues but do not control the cost of the product or service they sell or the level of investment in the responsibility center. In a revenue center, output is measured in monetary terms, but no formal attempt is made to relate input to output. Typically revenue centers are marketing/ sales units that do not have authority to set selling prices and are not charged for the cost of the goods they market. Actual sales orders booked are measured against budgets or quotas, and the manager is held accountable for the expenses incurred directly within the unit, but the primary measurement is revenue. Profit center: It is a responsibility center whose manager and other employees control both the revenues and the costs of the product or service they sell or deliver. Investment center: It is a responsibility center whose manager and other employees control the revenues, costs, and the level of investment in the responsibility center.

Q. What is Goal Congruence? What are the informal factors that influence Goal Congruence?Solu:-

Goal Congruence is a Central purpose of a management control system, then, it is to ensure a high level of what is called “Goal Congruence”.In a goal congruence process, the actions people are lead to take in accordance with their perceived self-interest are also in the best interest of the organization. Perfect congruence between individual goals and organizational goals does not exist.

Informal factors that Influence Goal Congruence:-Both formal systems and the informal process influence human behaviour in organizations. They

affect the degree to which goal congruence can be achieved. The formal control system consists of Strategic plans, budgets and reports. But it is important for the designer of the formal systems to take into account informal process such as work ethic, management style and culture. In order to implement the organization strategies effectively the formal mechanism must be consistent with the informal ones.

The factors are as follows:- 1) External Factors: External factors are norms of desirable behaviours that exist in the society of which the organization is a part. These norms includes a set of attitudes, often collectively referred to as the work ethic which is manifested in employees loyalties to the organization, their diligence, their spirit, their pride in doing a good job. Some of these attitudes are local i.e. city or rest on specific. Other attitudes and norms are industry specific.Example: Silicon Valley—a stretch of Northern California about 30 miles long and 10 miles wide—is one of the major source of new business creation and wealth in American economy.

2) Internal Factors: A. Culture:

The most important internal factor is organisation’s own culture—the common beliefs, shared values norms of behavior, and a assumptions that are implicitly accepted and explicitly manifested throughout

Page 30: mesmerizers.files.wordpress.com  · Web viewQ. Explain concept of Balance Scorecard. Ans: The Balanced Scorecard (BSC) is a performance management tool which began as a concept for

the organization. Cultural norms are extremely important since they explain why two organizations, with two identical formal management control systems, may vary in terms of actual controlExample: Johnson & Johnson has a strong corporate culture, as exemplified by the company’s credo. One cannot fully understand the effect of J&Js. Formal control system without considering the influence of their credo on the behaviour of its employees. This was demonstrated during the Tylenol crisis in 1982. After taking poisoned Tylenol capsules, 7 people died. J&J withdrew all Tylenol capsules from the US Market, even though all the poisoned capsules were sold in Chicago, the tampering occurred outside J&J premises, and the individual responsible was not the J&J employee. The steps were taken by the company to prevent such tampering in future.

B. Management Style: The internal factor that probably has the strongest impact on management control is management style. Usually, subordinates attitudes reflect what they perceive their superiors attitudes to be, and their superiors attitudes ultimately stem from the CEO. Managers come in all shapes and sizes. Some are charismatic and outgoing; others are less ebullient. Some spent much time looking and talking to people; other relies more heavily on written reports.

C. The Informal Organization: The lines on organization chart depict the formal relation -- the official authority and responsibilities of each manager. But in the course of fulfilling his responsibilities a manager interacts with many other people in the organization as well as other managers, support units, head quarters staff and those who are friends and acquaintances. This constitutes the informal organization. A manager sometimes may pay no head to the order from the formal boss because of this informal organization particularly if he is evaluated on the basis of functional efficiency.

D. Perceptions & Communications :In working towards the goals of the organization operating managers must know what the goals are and what action they are supposed to take in order to achieve them. They receive this information through various channels both formal and informal. Despite this range of channels, it is not always clear what senior management wants done. An organization is a complicated entity and the actions that should be taken by any one part to further the goals cannot be stated with absolute clarity.

Q. Briefly define Discretionary Expense Centers, Engineered Expense Center, and Profit Center & Investment Center? How is budget prepared in Discretionary Expense Center? How is performance of the manager evaluated in a Discretionary Expense Center?ANS) Discretionary Expense Centers including general and administrative (G&A) departments, such as finance, human resources, and legal; research and development (R&D) departments; and marketing units such as those performing advertising and promotion, are usually treated as discretionary expense centers. The output from these units is not easily measured in monetary terms, and the relationship between the resources they expend (inputs) and the outcomes they produce is weak. Companies control these discretionary expense centers by negotiating and eventually authorizing an annual budget and then monitoring whether their actual spending remains within the budgeted amounts.Engineered Expense Center: Engineered Expense Centers are usually found in manufacturing operations. Their input can be measured in monetary term. Their output can be measured in physical terms. The optimum dollar amount of input required to produce one unit of output can be determined. Manager of Engineered Expense Centers may be responsible for activities such as training & employee development that are not related to current production; their performance review should include an appraisal of how well they carry out their responsibilities.

Page 31: mesmerizers.files.wordpress.com  · Web viewQ. Explain concept of Balance Scorecard. Ans: The Balanced Scorecard (BSC) is a performance management tool which began as a concept for

Profit Center: Many operating unit managers have responsibility and authority for both production and sales. They make decisions about what products and services to produce, how to produce them, their quality level, price, sales and distribution systems. But these managers may not have the authority to determine the level of capital investment in their facilities. In these cases, operating profit may be the single best (short term) performance measure for how well the managers are creating value from the resources the company has put at their disposal. Such a unit, in which the manager has almost complete operational decision-making responsibility and is evaluated by a straight forward profit measure, is called a profit center.Investment Center: When a local manager has all the responsibilities described above as well as the responsibility and authority for his or her center’s working capital and physical assets, the manager is running an investment center. The performance of such a unit is best measured with a metric that relates profits earned to the level of physical and financial assets employed in the center. Investment center managers are evaluated with metrics as return on investment (ROI) and economic value-added.Budget prepared in Discretionary Expense Center:Management makes budgetary decisions for Discretionary Expense Centers that differ from those for engineered expense centers. Management formulates the budget for Discretionary Expense Center by determining the magnitude of the job that needs to be done. The work done by Discretionary Expense Centers falls into two general categories: continuing & special. Continuing work is done consistently from year to year, such as the preparation of financial statements by the controller’s office. Special work is a “one-shot” project—eg. Developing & installing a profit budgeting system in a newly acquired division.A technique often used in preparing a Discretionary Expense Center’s budget is Management by Objectives, a formal process in which a budgeted proposes to accomplish specific jobs & suggests the measurement to be used in performance evaluation. The planning function for Discretionary Expense Centers is usually carried out in one of two ways: incremental budgeting or zero-base review. Performance of the manager evaluated in a Discretionary Expense Center:The primary job of a Discretionary Expense Center’s manager is to obtain the desired output. Spending an amount that is “on budget” to do this is satisfactory; spending more than that is cause for concern; & spending less may indicate that the planned work is not done. In Discretionary Expense Centers the financial report is not a means of evaluating the efficiency of the manager. Total control over Discretionary Expense Centers is achieved primarily through no financial performance measures. E.g. the best indication of the quality of service for some Discretionary Expense Centers may be the opinion of their users.

Q. Every SBU is a profit center but every profit center is not a SBU? What are the conditions that should be fulfilled for an organization unit to be converted into a profit center? What are the different ways to measure the performance of profit center? Discuss their relative merits & demerits?ANS) Strategic Business Unit or SBU is understood as a business unit within the overall corporate identity which is distinguishable from other business because it serves a defined external market where management can conduct strategic planning in relation to products and markets. When companies become really large, they are best thought of as being composed of a number of businesses (or SBUs).

Page 32: mesmerizers.files.wordpress.com  · Web viewQ. Explain concept of Balance Scorecard. Ans: The Balanced Scorecard (BSC) is a performance management tool which began as a concept for

These organizational entities are large enough and homogeneous enough to exercise control over most strategic factors affecting their performance. They are managed as self contained planning units for which discrete business strategies can be developed. A Strategic Business Unit can encompass an entire company, or can simply be a smaller part of a company set up to perform a specific task. The SBU has its own business strategy, objectives and competitors and these will often be different from those of the parent company.

Profit Centres are parts of a Corporation that directly add to its Profit. A profit center manager is held accountable for both revenues, and costs (expenses), and therefore, profits. What this means in terms of managerial responsibilities is that the manager has to drive the sales revenue generating activities which leads to cash inflows and at the same time control the cost (cash outflows) causing activities.Profit center management is equivalent to running an independent business because a profit center business unit or department is treated as a distinct entity enabling revenues and expenses to be determined and its profitability to be measured. Business organizations may be organized in terms of profit centers where the profit center's revenues and expenses are held separate from the main company's in order to determine their profitability. Usually different profit centers are separated for Accounting purposes so that the management can follow how much profit each center makes and compare their relative efficiency and profit. After learning about the profit center & SBU, hence we can say that every SBU is a profit center but every profit center is not a SBU.The conditions that should be fulfilled for an organization unit to be converted into a profit center are:

The manager should have access to the relevant information needed for making such a decision. There should be some way to measure the effectiveness of the trade-offs the manager has made.

There are 2 types of measuring the performance of the profit center they are: The measure of Management Performance, which focuses on how well the manager is doing.

This measure is used for planning, coordinating, & controlling the profit center’s day-to-day activities & as a device for providing the proper motivation for its manager.

The measure of Economic Performance, which focuses on how well the profit center is doing as an economic entity. A profit center’s economic performance is always measured by net income ( i.e., the income remaining after all costs, including a fair share of the corporate overhead, have been allocated to the profit center). The performance of the profit center manager, however, may be evaluated by 5 different measures of profitability & they are:

1. Contribution Margin,2. Direct profit,3. Controllable Profit,4. Income before Income Taxes, or5. Net Income.

Merits of Profit Centers:1) The quality of decisions may improve because they are being made by managers closest to the point of decision.

Page 33: mesmerizers.files.wordpress.com  · Web viewQ. Explain concept of Balance Scorecard. Ans: The Balanced Scorecard (BSC) is a performance management tool which began as a concept for

2) The speed of operating decisions may be increased since they do not have to be referred to corporate headquarters.3) Headquarters management, relieved of day-to-day decision-making, can concentrate on border issues. 4) Managers, subject to fewer corporate restraints, are freer to use their imagination & initiative.5) Profit centers provide an excellent training ground for general management & for managers.6) Profit Consciousness is enhanced since managers who are responsible for profits will constantly seek ways to increase them.7) Profit centers provide top management with ready-made information on the profitability of the company’s individual components.8) As the output is readily measured, profit centers are particularly responsive to pressures to improve their competitive performance.

Demerits of the Profit Centers:1) Decentralized decision-making will force top management to rely more on management control

reports than on personal knowledge of an operation, entailing some loss of control.2) If headquarters management is more capable or better informed than the average profit center

manager, the quality of decisions made at the unit level may be reduced.3) Friction may increase because of arrangement over the appropriate transfer price, the assignment

of common costs, & the credit for revenues that were formerly generated jointly by two or more business units working together.

4) Organization units that once cooperated as functional units may now be in competition with one another. An increase in profits for one manager may mean a decrease for another. In such situations, a manager may fail to refer sales leads to another business unit better qualified to pursue them; may hoard personnel or equipment that, from the overall company standpoint, would be better off used in another unit; or may make production decisions that have undesirable csot consequences for other units.

5) Divisionalization may impose additional costs because of additional management. Staff personnel and record keeping required, and may lead to task redundancies at each profit center.

6) Competent general managers may not exist in a functional organization because there may not have sufficient opportunities for them to develop general management competence.

7) There is no completely satisfactory system for ensuring that optimizing the profits of each individual profit center will optimize the profits of the company as a whole.

8) There may be too much emphasis on short-run profitability at the expense of long-run profitability. In the desire to report high current profits, the profit center manager may skimp on R&D, training prog, or maintenance. This tendency is especially prevalent when the turnover of profit center managers is relatively high. In these circumstances, managers may have good reason to believe that their actions may not affect profitability until after they have moved to other jobs.

Q. Write Short notes on:

Page 34: mesmerizers.files.wordpress.com  · Web viewQ. Explain concept of Balance Scorecard. Ans: The Balanced Scorecard (BSC) is a performance management tool which began as a concept for

I. Zero-based budgeting Zero based budgeting is a technique of planning and decision-making which reverses the working process of traditional budgeting. In traditional incremental budgeting, departmental managers justify only increases over the previous year budget and what has been already spent is automatically sanctioned. No reference is made to the previous level of expenditure. By contrast, in zero-based budgeting, every department function is reviewed comprehensively and all expenditures must be approved, rather than only increases. Zero-based budgeting requires the budget request be justified in complete detail by each division manager starting from the zero-base. The zero-base is indifferent to whether the total budget is increasing or decreasing. Zero-based budgeting starts from a “zero base” and every function within an organization are analyzed for its needs and costs.Advantages of Zero-Based Budgeting

1. Efficient allocation of resources, as it is based on needs and benefits. 2. Drives managers to find cost effective ways to improve operations. 3. Detects inflated budgets. 4. Municipal planning departments are exempt from this budgeting practice. 5. Useful for service departments where the output is difficult to identify. 6. Increases staff motivation by providing greater initiative and responsibility in decision-making. 7. Increases communication and coordination within the organization. 8. Identifies and eliminates wasteful and obsolete operations. 9. Identifies opportunities for outsourcing. 10. Forces cost centers to identify their mission and their relationship to overall goals.

Disadvantages of Zero-Based Budgeting1. Difficult to define decision units and decision packages, as it is time-consuming and exhaustive. 2. Forced to justify every detail related to expenditure. The R&D department is threatened whereas

the production department benefits. 3. Necessary to train managers. Zero-based budgeting must be clearly understood by managers at

various levels to be successfully implemented. Difficult to administer and communicate the budgeting because more managers are involved in the process.

4. In a large organization, the volume of forms may be so large that no one person could read it all. Compressing the information down to a usable size might remove critically important details.

5. Honesty of the managers must be reliable and uniform. Any manager that exaggerates skews the results.

II. Free Cash FlowA measure of financial performance calculated as operating cash flow minus capital expenditures. Free cash flow (FCF) represents the cash that a company is able to generate after laying out the money required to maintain or expand its asset base. Free cash flow is important because it allows a company to pursue opportunities that enhance shareholder value. Without cash, it's tough to develop new products, make acquisitions, pay dividends and reduce debt. FCF is calculated as:

Page 35: mesmerizers.files.wordpress.com  · Web viewQ. Explain concept of Balance Scorecard. Ans: The Balanced Scorecard (BSC) is a performance management tool which began as a concept for

It can also be calculated by taking operating cash flow and subtracting capital expenditures. If free cash flow is positive then the company has done a good job of managing its cash. If free cash flow is negative then the company may have to look for other sources of funding such as issuing additional shares or debt financing.If a company has a negative free cash flow and has to issue more equity shares, this will dilute the profits per share. If the company chooses to seek debt financing, there will be additional interest expense as a result and the net income of the company will suffer.When investing for dividend growth, we can assume that for a company to continuously grow its dividend there must be positive cash flow.Free cash flow is one indicator of the ability of a company to return profits to shareholders through debt reduction, increasing dividends, or stock buybacks. All of these scenarios result in an increased shareholder yield and a better return on investment. The term “Free cash Flow” is used because this cash is free to be paid back to the suppliers of capital.

III. Management Control in Matrix OrganizationIf members of the project team are employees of the sponsoring organization, they have 2 bosses, the project manager & the manager of the functional department to which they are permanently assigned. Such an arrangement is called Matrix Organization. However, their basic loyalty is to their functional department. Therefore, project manager has less authority over personnel than the manager of a production department, whose employees have an undivided loyalty to that department.Project managers want full attention given to their projects, while functional responsibility center managers must take into account all the projects on which the employees of that center work. This conflict of interest is inevitable; it creates tension. Hence it is necessary to have a good management control on this type of organization.

An evolution of organization structure should be in place to handle the work smoothly. Different types of management personnel & management methods may be appropriate at different stage of the project. In the planning phase of a construction project, architects, engineers, schedulers, & cost analysts predominate. In the execution of the project, the managers are production managers. In the final stages, the work tapers off, & the principal task may be to obtain the sponsor’s acceptance, with marketing skills being a principal requirement.If the project is conducted by an outside contractor, an additional level of project control is created. In addition to the control exercised by the contractor who does the work, the sponsoring organization has its own control responsibilities. The contractor could bring its own control system to the project, & this system need to be adopted to provide information that the sponsors needs. This doesn’t imply that there are duplicate systems; the sponsor’s system should use data from the project system.

Page 36: mesmerizers.files.wordpress.com  · Web viewQ. Explain concept of Balance Scorecard. Ans: The Balanced Scorecard (BSC) is a performance management tool which began as a concept for

IV. Internal ControlInternal control is the process by which managers influence other members of the organization to implement the organization’s strategies. Internal control involves a variety of activities, including:

Planning what the organization should do. Coordinating the activities of several parts of the organization. Communicating information. Deciding what, if any, action should be taken. Influencing people to change their behavior.

Internal control doesn’t necessarily require that all actions correspond to a previously determined plan, such as a budget. Such plans are based on circumstances believed to exist at the time they were formulated. If these circumstances have changed at the time of implementation, the actions dictated by the plan may no longer be appropriate.

Q. investment centre(I) Introduction to investment centre:An investment centre is responsible for both profits and investments. The investment centre manager has control over revenues, expenses and the amounts invested in the centre’s assets. He also formulates the credit policy which has a direct influence on debt collection, and the inventory policy which determines the investment in inventory. The manager of an investment centre has more authority and responsibility than the manager of either a cost centre or profit centre. Besides controlling costs and revenues, he has investment responsibility too. Investment on asset responsibility means the authority to buy, sell and use divisional assets.

(II) Definition:Department or an area of responsibility, where a manager controls revenues and associated costs, assets, and liabilities. His or her performance is assessed largely on the basis of return on investment (ROI) achieved.

(III) Methods of evaluating performance of an investment centre:Performance measurement is aimed in aimed in respect of all responsibility centres. Whether dealing with a cost profit or investment centre, performance can be evaluated by comparing results with something. Probably the best way to encourage managers to act in the firm’s best interests is to measure their performance of a division:

1. Variance analysis:Variance analysis may be:

(i) Cost variance analysis and (ii) Revenue variance analysis

Performance of a cost centre can be evaluated using cost variance analysis, i.e., by comparing the actual costs with the standard costs. Standard cost display the costs the cost centre should have incurred, given

Page 37: mesmerizers.files.wordpress.com  · Web viewQ. Explain concept of Balance Scorecard. Ans: The Balanced Scorecard (BSC) is a performance management tool which began as a concept for

their actual activity. Any variance between the actual costs and standard cost require corrective action by the management. On measuring the performance attempts are made to minimize the costs of cost centre.

Revenue variance analysis can be used in evaluating the performance of a revenue centre. If actual sales (revenues) are different from the budgeted sales, they become the primary focus of the management attention. While evaluating the performance of revenue centre, costs incurred are not used as yardstick.

2. Division contribution Margin: Division contribution margin is defined as the total division revenue less the direct cost of the division. The measure of performance emphasizes the contribution of each division to overall company profit. In this measure, all revenues and costs are traceable to the division are included and all common, indirect costs of the company are excluded.

Top management of the company often use the direct contribution margin to decide in which divisions additional investments should be made and in this decision will surely prefer those divisions which are giving greatest contribution margin.

3. Division Net Profit:The division net profit is the most appropriate profit measure for evaluation of a division’s performance. This analysis can be used to judge the performance of a profit centre as both cost and revenue data measured in financial terms are available.

It is claimed that while measuring the performance of a profit division, it is better to use a measure of divisional revenue less divisional costs and divisional revenue less divisional controllable costs should be used to measure the performance of a divisional manager.

4. ROI:This measure expresses divisional profit as a percentage of the firm’s investment in the division and is similar to the widely accepted ‘return on capital employed’ measure used in the external analysis and interpretation of accounts.This is calculated as: The first part of the ratio is called Profit margin and second part is called Asset turnover ratio.

Advantages:(i) It relates net income to investment made in a division giving a better measure of divisional

profitability.(ii) It can be used as a basis for other ratios which are useful for analytical purposes.(iii) It is easy to understand as it is based on financial accounting measurements.

ROI = = *

Divisional Profit Divisional InvestmentNet Profit Sales

Sales Investment

Page 38: mesmerizers.files.wordpress.com  · Web viewQ. Explain concept of Balance Scorecard. Ans: The Balanced Scorecard (BSC) is a performance management tool which began as a concept for

RI = Divisional Profit – (Percent charge * Divisional investment)

(iv) It may be used for inter-firm comparision, provided that the firms whose results are being compared are of comparable size and of the same industry.

Dis-advantages:(i) Satisfactory definition of profit and investment are difficult to find. Profit has many concepts such as

PBIT, PAT, controllable profit. Similarly, the term investment may have many conations such as gross book value, net book value, historical cost of assets, current cost of asset, asset including or excluding intangible assets.

(ii) While comparing ROI of different companies it is necessary that the companies use similar accounting policies and methods in respect of valuation of stocks, valuation of fixed assets, apportion of overheads, treatment of research and development expenditure.

(iii) ROI may influence a divisional manager to select only investments with high rates of return. Other investments that would reduce ROI but could increase the value of business may be rejected by the divisional manager.

5. Residual Income:Residual income can be defined as the net income of a division, less the ‘imputed’ capital charge on the assets used by the division. The capital charge is the minimum acceptable rate of return and is calculated by applying this required rate of return to the division’s investment base. RI is calculated as:

Advantages:(i) It avoids suboptimal decisions as investments are not rejected merely because they lower divisional

manager’s ROI.(ii) It maximizes growth of the company and increases shareholder’s wealth by accepting opportunities

which earn a rate of return in excess of the cost of capital.(iii) The cost of capital charge on divisional investments ensures that the divisional managers are aware of

the opportunity cost of funds.(iv)Charging each division with the company’s cost of capital ensures that decisions taken by different

divisions are compatible with the interests of the organization as awhole.

Dis-advantages:

(i) Like ROI, it is difficult to have satisfactory definition of ‘divisional profit’ and ‘divisional investment’.(ii) It may be difficult to calculate an accurate cost of capital. Also decision has to be taken to use

company’s cost of capital or specific divisional cost of capital. The former enhance divisional goal congruency and the later reflects each division’s level of risk.

(iii) Identifying controllable and uncontrollable factors at divisional level may be difficult.Q. PROFESSIONAL ORGANISATIONS AND THEIR CHARACTERISTICS::

Page 39: mesmerizers.files.wordpress.com  · Web viewQ. Explain concept of Balance Scorecard. Ans: The Balanced Scorecard (BSC) is a performance management tool which began as a concept for

Professional organization (also called a professional body, professional association or professional society) is an organization seeking to further a particular profession, the interests of individuals engaged in that profession, and the public interest.The roles of these professional associations have been variously defined: "A group of people in a learned occupation who are entrusted with maintaining control or oversight of the legitimate practice of the occupation;" also a body acting "to safeguard the public interest; organizations which "represent the interest of the professional practitioners," and so "act to maintain their own privileged and powerful position as a controlling body."

Such bodies generally strive to achieve a balance between these two often conflicting mandates. Though professional bodies often act to protect the public by maintaining and enforcing standards of training and ethics in their profession, they often also act like a cartel or a labor union (trade union) for the members of the profession, though this description is commonly rejected by the body concerned.

Therefore, in certain dispute situations the balance between these two aims may get tipped more in favor of protecting and defending the professionals than in protecting the public. An example can be used to illustrate this. In a dispute between a lawyer and his/her client or between a patient and his/her doctor, their remains a conflict of interest in (a) its wish to defend the interests of the client, while also (b) wishing to defend the interests, status and privileges of the professional. It is clearly a tough call for it do both.Organizations that provide professional services are:

Law firms Consulting firms Engineering firms Sports organization Hospitals

CHARACTERISTICS OF PROFESSIONAL ORGANIZATION1. SMALL SIZE

Most professional organizations are small in size and operate in a single location, with the exception of law and accounting firms. Senior management personally monitors and motivates their subordinates. This brings down the need for sophisticated control systems. Even though professional organizations are small, they still need to prepare budgets, compare the budgeted and actual performance, and compensate employees on the basis of their performance.

2. LABOUR INTENSIVENESS Professional organizations are labour intensive and they employ individuals who are specialist in respective fields. Professionals, who are also managers, prefer working independently rather than in teams. Professionals are the most valuable asset of an organization. Due to this, some management thinkers advocate the idea that these professionals should be valued highly and their value highly included in the company’s balance sheet. A system called human resource accounting was developed by Likert for valuing the professionals but very few companies used it.

Page 40: mesmerizers.files.wordpress.com  · Web viewQ. Explain concept of Balance Scorecard. Ans: The Balanced Scorecard (BSC) is a performance management tool which began as a concept for

3. DIFFERENT OBJECTIVES AND GOALS The goal of manufacturing organization is to earn satisfactory return on the assets it employed. As more assets are tangible, they appear in the balance sheet. In professional organizations, as most assets assume the form of the employees skills, it is difficult for the company to set for itself a goal in terms of returns on assets employed. The financial goal of professional organizations is to provide satisfactory compensation to its employees. Another goal of professional organizations is to increase their sizes and networks.

4. DIFFICULTY IN MEASURING OUTPUT The output of manufacturing organizations can be measured in terms of units, tons, gallons, etc but this method cannot be applied to professional organizations. For example, the output of physician, can be measured in terms of number of patients treated but one cannot measure whether the service provided by the physician satisfy the patient. In some cases, the revenues earned indicate the measure of output but only in terms of quantity. In a professional service organization, the non-repetitive nature of work compounds the problem of measuring output. Since no two professionals work in the same way, it is difficult to set standards in terms of the time spent for a task and the way in which the task is performed.

5. DIFFERENT MARKETING STRATEGY (How marketing is done in professional organization?) In manufacturing companies, production and marketing activities are clearly demarcated. But no such demarcation is done in professional organizations. These organizations do not market themselves openly. It is done through the use of articles, personal and professional contacts, speeches, etc. an auditing firm may market itself through the articles written by its auditors (on contemporary issues) or through the marketing activities done by the professionals who spent much of their time working for clients. Thus, it becomes difficult to identify a small employee who is responsible for promoting the organization.

PERFORMANCE EVALUATION OF PROFESSIONALSPerformance of professionals can be evaluated by considering the following criteria: Key Responsibilities and DutiesDescribe the major responsibilities. For each one, identify the most significant tasks and duties involved which will have the most importance in determining the employee’s overall performance level. Scope of Control/ResponsibilityDescribe the span of control in terms of budget, size of staff, impact of decisions and other relevant indices. Knowledge and AdaptabilityUnderstanding and use of assignments; demonstrated confidence and professionalism; willingness to grow in a position. Leadership

Page 41: mesmerizers.files.wordpress.com  · Web viewQ. Explain concept of Balance Scorecard. Ans: The Balanced Scorecard (BSC) is a performance management tool which began as a concept for

Effectiveness in directing, motivating, and evaluating the work of staff in the unit; quality of development assistance provided employees for professional growth; effectiveness in delegating and controlling work of subordinates. Interpersonal Skills and CommunicationContribution to cooperative relationships with other university departments, students and the public; effectiveness of written and verbal communication; ability to create a positive environment; ability to build teamwork and cooperation with others within the outside of the department. Planning and Organizational SkillsSuccess in planning, implementing and monitoring new and existing methods, procedures and programs; ability to set and achieve realistic objectives and to determine priorities; effectiveness in use of time; achievement of position deadlines. Judgment and InitiativeResourcefulness in accomplishing and solving problems; ingenuity in finding new solutions; ability to function with minimal supervision and control; decision making ability; desire for new responsibilities; ability to make program contributions and develop new methods and procedures. Financial Management and ControlAbility to analyze and control expenditures with appropriate concern for cost control; effective use of available resources and maintenance of budget within prescribed limits. Problem Solving/Decision Making SkillsUnderstanding factors in developing sound practical solutions; making prompts decisions, accepting responsibility; making creative contributions to solution of problems and resolution of disputes. Supervision ReceivedDescribe the level of supervision over this position in terms of decisions, authority level, expected amount of independent decision making, judgment and initiative required, etc. Professional DevelopmentEfforts to continue professional development in a specialized area of skill or expertise; research activities; publications; attendance at professional conferences; workshops and seminars, presentation of papers at professional meetings, conferences, etc.

Q. What are different types of strategic missions at sbu level ? how do these missions affect strategic planning process and budgeting at sbu levelStrategic Business Unit or SBU is understood as a business unit within the overall corporate identity which is distinguishable from other business because it serves a defined external market where management can conduct strategic planning in relation to products and markets. When companies become really large, they are best thought of as being composed of a number of businesses (or SBUs).These organizational entities are large enough and homogeneous enough to exercise control over most strategic factors affecting their performance. They are managed as self contained planning units for which discrete business strategies can be developed. A Strategic Business Unit can encompass an entire company, or can simply be a smaller part of a company set up to perform a specific task. The SBU has its own business strategy, objectives and competitors and these will often be different from those of the parent company. Research conducted in this include the BCG Matrix.

Page 42: mesmerizers.files.wordpress.com  · Web viewQ. Explain concept of Balance Scorecard. Ans: The Balanced Scorecard (BSC) is a performance management tool which began as a concept for

This approach entails the creation of business units to address each market in which the company is operating. The organization of the business unit is determined by the needs of the market.An SBU is an operating unit or planning focus that groups a distinct set of products or services, which are sold to a uniform set of customers, facing a well-defined set of competitors. The external (market) dimension of a business is the relevant perspective for the proper identification of an SBU. Therefore, an SBU should have a set of external customers and not just an internal supplier. (Arnoldo C. Hax and Nicholas Majluff)

A Strategic Business Unit ( SBU ) is generally defined by what it has in common, as well as the traditional aspects defined by McKinsey, of separate competitors and a profitability bottom lineThe commonalities are five in number:An SBU may be defined by its common raw materials. Such a company, Kimberly-Clark for example has a variety of paper products such as Kleenex, Kotex (sanitary napkins ), Huggies ( originally based on cellulose).

Another SBU may be defined by its common manufacturing processes: A company such as General Motors may have separate units making automobiles, trucks, diesel-electric locomotives, all based on such engineering and assembly processes.

Another SBU relies on a common distribution method; “The Sharper Image” started by utilizing advertisements only in airline magazines using direct mail . Its success eventually brought it to using regular retailing stores as well

Yet another SBU is based on common customers: Proctor & Gamble makes Pringles potato chips, Ultra-Pampers, Dawn detergent, Crest toothpaste, all based on the fact that these products are largely sold through super markets. In the past several years they also acquired Gillette Razors for much the same reason.A last SBU may be based on its service processes, such as an accounting firm ( Deloitte Touche ) or a consulting firm ( Accenture ) where its particular set of skills of its personnel have a clear competitive advantage. A shipper/ warehouser such as UPS or Federal Express similarly fits this definition.There are three factors that determine the success of an SBU: 1. The degree of autonomy given to each SBU manager 2. The degree to which an SBU shares functional programs and facilities with other SBU's 3. The manner in which the corporation evaluates and rewards the performance of its SBU managersCompanies today often use the word “Segment” or “Division” when referring to SBU’s, or an aggregation of SBU’s that share such commonalities.

STRATEGIC OBJECTIVESThere is no one-size-fits-all answer for the effective design of control systems. When an SBU manager's reward system is matched with the SBU's strategy, performance will match corporate strategy, and objectives will be enhanced. Failure to match strategy and reward will adversely affect the manager's

Page 43: mesmerizers.files.wordpress.com  · Web viewQ. Explain concept of Balance Scorecard. Ans: The Balanced Scorecard (BSC) is a performance management tool which began as a concept for

motivation and efforts. The strategy for an SBU is dependent on its mission and the consideration of environmental opportunities, internal strengths, and the resources available to accomplish its goals and objectives. (2) Three approaches to control system design that foster goal congruence are: situation specific, universalistic, and contingency. (3) The situation-specific model views each situation as unique, so application of general rules is not possible. Universalists argue that an optimal control system design will be effective in all settings. The contingency approach, which has become the prominent paradigm, is positioned between these two extremes. It suggests that the appropriateness of the control system depends on the business setting (like the situation-specific approach), but generalization (universalistic approach) can be made across similar settings. If the SBU mission or competitive strategy varies across divisions within the organization, the control system must be modified to capture the relevant performance measures and motivate SBU managers accordingly. (4) Strategic mission or business unit strategies are commonly grouped into the following areas: build, hold, harvest, and divest. (5) Competitive strategies include: low cost, differentiation, focus, defender, or prospector. (6)A Strategic Business Unit (SBU) has been defined above in the strategic planning process. Each of a firm’s

Strategic Business Units (SBU) has six attributes:1. A specific target market2. Its own senior marketing executive3. Control over its resources4. Its own marketing strategy5. Clear-cut competition6. Distinct differential advantages

STRATEGIC MISSIONThe mission of an international SBU is related to lifecycle concepts. A build mission implies the goal of increasing market share and typically applies to any SBU with low market share in a high-growth industry. In order to build a competitive advantage, it may be necessary for the manager to sacrifice short-term earnings and/or cash flow. Also, a build strategy implies an increase in production, which results in additional use of the firm's resources. Performance measures that focus mainly on profit or return would be in conflict with the overall mission of the SBU. The manager, therefore, should be evaluated and rewarded primarily on achieving a targeted increase in sales or market share, with profitability measures (with a great deal of slack) a secondary objective.

The hold strategy applies to an SBU with a high market share in a high-growth industry. Though profit-oriented accounting performance measures would be appropriate, nonfinancial measures also should be incorporated, such as customer service, maintaining market share, and quality measures.

The goal of a harvest SBU is to maximize short-term cash flows and earnings at the expense of market share (high market share, low-growth industry). These earnings can then supplement other business units that may be in build strategies. To align management decision making with the harvest strategy, the

Page 44: mesmerizers.files.wordpress.com  · Web viewQ. Explain concept of Balance Scorecard. Ans: The Balanced Scorecard (BSC) is a performance management tool which began as a concept for

control system should evaluate performance using one of the conventional return measures, such as ROI, RI, or EVA. Measures of cash flow also may be appropriate. Profit and return measures have a much tighter acceptable range and should be adhered to strictly.

In a low-market-share, low-growth industry, the SBU's strategy may be to divest through a process of slow withdrawal or outright sale. The appropriate control system at this stage is unique to the particular situation. Presumably, the objective is to maximize cash flows. This strategy represents the end of the life-cycle stages, however, and is a unique situation with a limited ability to generalize. Therefore, we will skip the discussion of control models for the divest strategy.

Certain objectives of SBU’sa. focus on market share and/or sales growthb. maintain market sharec. new products to marketd. quality and/or customer service measurese. cost management/efficiency measuresf. return-based measures (e.g.,ROI,EVA)g. tight budgetary controlsh. slack in budgetary controlsi. group-based rewards/evaluationj. individual-based rewardsk. preference for evaluation relative to othersl. business-unit vs. company rewardsm. company-based vs. business-unit rewardsn. formula-based evaluation/rewards/bonuseso. subjective evaluation/rewards/bonusesp. pay for performance/contingent rewardsq. performance-based rewards less motivatingr. desire for incentive-based extrinsic rewardss. intrinsic rewards likely to be valuedt. focus on short-term financial performanceu. past/present orientationv. future orientation/long planning horizonsw. preference for immediate rewardsx. motivated by deferred compensationy. acceptance/desire for stretch budgetsz. preference for interactive budget process

MISSION AFFECT STRATEGIC PLANNING PROCESSAn SBU with a low-cost competitive strategy attempts to achieve lower costs relative to competitors. Typical low-cost actions include taking advantage of economies of scale; learning-curve effects; reducing

Page 45: mesmerizers.files.wordpress.com  · Web viewQ. Explain concept of Balance Scorecard. Ans: The Balanced Scorecard (BSC) is a performance management tool which began as a concept for

customer service, research and development, advertising, and/or sales force; and maintaining a stable product line. Strict adherence to cost standards through variance analysis and other measures of operating efficiency, such as cycle time and inventory turnover, would be appropriate measures to evaluate and control performance in this competitive environment. Further, A. A. Thompson and A. J. Strickland, III, suggest that significant cost advantages can emerge from an analysis of an entity's internal and external value chain. (7) Low-cost participants must be careful, however, because the marketplace will still demand a minimum level of product quality and functionality.

A differentiation strategy focuses the SBU manager's attention on brand loyalty, customer service, product design, and technology. The goal is to create a product that customers view as unique and exclusive. Product innovation is critical. To create uniqueness, the SBU is likely to have a more diversified set of products or functionally superior products compared to a low-cost competitor, and it must invest in research and product development, technology, marketing, and customer service. Achieving a target ROI does not measure progress effectively within this strategy. While traditional financial performance measures still play a role, nonfinancial performance measures, such as quality, on-time deliveries, customer satisfaction, and number of new products to market, must be emphasized.

An SBU with a focus strategy targets a narrow competitive market within an industry segment. The specific objectives could be either low cost or differentiation. Design of the control system must be tailored to the selected objective.

SBUs with a defender strategy engage in limited product/market research, have limited product lines, and have a stable environment. Defenders compete through cost and quality control. This strategy is consistent with the features of the hold and harvest missions. ROI, RI, and EVA may be effective control measures if they are incorporated with variance analysis, operating efficiency measures, and quality variables.

Prospector SBUs, similar to differentiators, compete by focusing on market development, new product development, and searching (prospecting) for market opportunities. These SBUs are often in a build strategic mission. Profit-oriented performance measures would not capture progress toward goals and objectives. The number of new products to market, customer satisfaction, quality, sales from products developed in the last 24 months, and market share would evaluate and control the manager more appropriately.

As we said, there is no single performance measure or control system applicable across different business unit strategies that provides a basis for aligning management decision making with corporate goals. The specific control system must be modified and aligned with the particular strategy of the SBU. The preferences of the SBU manager in the design of the control system, therefore, must not be ignored. For example, the preferences of international SBU managers for autonomy, level of uncertainty, risk, participation, group versus individual rewards, and short- versus long-term rewards may be influenced by

Page 46: mesmerizers.files.wordpress.com  · Web viewQ. Explain concept of Balance Scorecard. Ans: The Balanced Scorecard (BSC) is a performance management tool which began as a concept for

their cultural identity. In turn, consideration of cultural differences in the design of a control system can increase its effectiveness.

Also affects certain cultural factorsThese days, more and more MNCs use local management talent to operate a foreign subsidiary rather than relying on "imported" expatriates. Effectiveness of the management control system depends on whether the local manager of the SBU perceives the control system as aligned with the shared values maintained in the host country. Hofstede defined culture as "the collective programming of the mind that distinguishes the members of one category of people from those of another." In perhaps the most extensive and most frequently cited research conducted with respect to cultural differences, Hofstede identified five underlying cultural dimensions--power distance, uncertainty avoidance, individualism versus collectivism, masculinity versus femininity, and Confucian dynamism--and assigned scores on them to more than 50 countries.

The business units (or products) on the basis of their relative market shares and growth rates. Cash cows are units with high market share in a slow-growing industry. These units typically

generate cash in excess of the amount of cash needed to maintain the business. They are regarded as staid and boring, in a "mature" market, and every corporation would be thrilled to own as many as possible. They are to be "milked" continuously with as little investment as possible, since such investment would be wasted in an industry with low growth.

Dogs, or more charitably called pets, are units with low market share in a mature, slow-growing industry. These units typically "break even", generating barely enough cash to maintain the business's market share. Though owning a break-even unit provides the social benefit of providing jobs and possible synergies that assist other business units, from an accounting point of view such a unit is worthless, not generating cash for the company. They depress a profitable company's return on assets ratio, used by many investors to judge how well a company is being managed. Dogs, it is thought, should be sold off.

Question marks (also known as problem child) are growing rapidly and thus consume large amounts of cash, but because they have low market shares they do not generate much cash. The result is a large net cash consumption. A question mark has the potential to gain market share and become a star, and eventually a cash cow when the market growth slows. If the question mark does not succeed in becoming the market leader, then after perhaps years of cash consumption it will degenerate into a dog when the market growth declines. Question marks must be analyzed carefully in order to determine whether they are worth the investment required to grow market share.

Stars are units with a high market share in a fast-growing industry. The hope is that stars become the next cash cows. Sustaining the business unit's market leadership may require extra cash, but this is worthwhile if that's what it takes for the unit to remain a leader. When growth slows, stars become cash cows if they have been able to maintain their category leadership, or they move from brief stardom to dogdom.

Uncertainty avoidance refers to the society's preference for risk-free, unambiguous situations. A culture with a high uncertainty-avoidance score (Greece, Guatemala, Japan, Chile, Argentina, Spain) reflects a

Page 47: mesmerizers.files.wordpress.com  · Web viewQ. Explain concept of Balance Scorecard. Ans: The Balanced Scorecard (BSC) is a performance management tool which began as a concept for

preference for control systems that adhere to clearly defined performance measures with unambiguous links to performance evaluation and reward. A low uncertainty-avoidance culture (Singapore, Hong Kong, Sweden, U.K., Malaysia), on the other hand, is more open to less structured control systems where rewards are either discretionary or include a bonus scheme rather than fixed compensation. With a preference for uncertainty avoidance, security in one's position is paramount and rigid, and specific rules that reduce uncertainty are generally accepted. Ambiguity in the control system may be perceived as a continuous threat and result in low morale, high turnover, and increased stress and anxiety.In Hofstede's research, there appears to be an interaction of power distance and uncertainty avoidance. Though the surveyed countries scatter across all four quadrants, there is a significant grouping of countries in the low power distance, low uncertainty avoidance sector (Sweden, U.K., U.S., Canada) and in the high power distance, high uncertainty avoidance quadrant (Spain, Mexico, Argentina, Chile, Brazil). (12) The countries in the former group prefer participation in the budget process, but they also would accept increased risk in the reward structure. Countries in the latter group represent cultures that accept inequality in the power structure and a lack of participative budgets, but they would demand certain fixed rewards.

Individualism versus collectivism represents the degree that members of a society perceive themselves as individuals rather than as members of a group. Countries with a high score on this dimension (U.S., U.K., Canada, New Zealand, Italy, Sweden) reflect a culture with preferences for individual versus group rewards, independence, and recognition of personal achievement. Tight budgetary controls can be perceived as stifling individual performance. In contrast, a control system with slack budgetary controls and individual rewards imposed on a collectivist culture (Guatemala, Singapore, Hong Kong, Malaysia, Mexico, Greece, Brazil, Chile) would be perceived as contrary to societal norms. Such a system highlights individual differences and promotes competition and interpersonal rivalries, contrary to a preference for group harmony and equality.

Masculinity versus femininity indicates the extent that the "masculine" values of assertiveness, ambition, independence, competitiveness, and male dominance are revered over the "feminine" values of nurturing, interdependence, service motivation, quality of life, and equality between sexes. In Hofstede's research, the higher a country scored on masculinity, the greater the gap between the values of its men and women. In a high-masculinity country (Japan, Austria, Mexico, Germany, U.K.), managers are more accepting of stretch budgets and would prefer a focus on individual achievement with performance evaluated relative to peers. In a high-masculinity country such as Japan, the high Confucianism factor (discussed next) and relatively low individualism score may mitigate the desire for individual recognition. In addition, anecdotal evidence suggests that managers of SBUs in some high-masculinity countries may be open to stretch budgets, but when they fail to deliver they still expect their fixed reward. In low-masculinity countries (Sweden, Chile, Spain, Singapore, Brazil, Canada), a strong emphasis on bottom-line profitability with little concern for members of the organization will be met with resistance and is therefore counterproductive to achieving SBU goals. Quality-of-life values such as a friendly work environment, cooperation, and intrinsic rewards are likely to be motivational factors combined with a preference for group performance evaluation versus individual, incentive-based rewards.

Page 48: mesmerizers.files.wordpress.com  · Web viewQ. Explain concept of Balance Scorecard. Ans: The Balanced Scorecard (BSC) is a performance management tool which began as a concept for

STRATEGIC PLANNING PROCESS AFFECTS BUDGETING PREPARATIONSBudgets are an important tool for effective short term planning and control in organisatons, an operating budger usually overs one year and states the revenues and expensed planned for that yearThe central function achieved by the Budget at the preparation stage is resource allocation. Not the whole economy resources, but those commanded by the Government. The role of Government is not only determination of budget appropriation, but also setting policies for their determination. Bringing consistency in the picture at the Budget Preparation stage requires planning, where a medium to long-range view of future is taken and all policies envisaged are brought together. They role is not only determination of budget appropriation, but also setting policies for their determination. To bring consistency in the picture at the Budget Preparation stage requires planning, where a medium to long-range view of future is taken and all policies envisaged are brought together. International experience shows that the integration of policy, planning and budgeting is an essential factor contributing to good budget outcomes at the macroeconomic, strategic and operational levels. They ensure that expenditure programs are driven by policy priorities and disciplined by budget realities. 

Being compatible with international standards and practices Reduce paperwork and streamline the budgeting processes Being capable of producing extensively detailed and accurate budgets based on accurate

information – past, current and future projections Definition of budget structure and budgetary periods Supporting program/performance budgeting

o Matching appropriations and budget execution to projects and performance criteria o Variance analysis and future development prognoses

Supporting multi-year budget planning Allowing budget scenarios and "what-if" analysis Allowing „Top-Down“ and „Bottom-Up“ budget  preparation Being capable of producing reports required locally and internationally on time and in required

format

Expenditure Management is used for realization of the Treasury client’s (non-profit, contributory or other public organizations) expenditure and revenue cycle (from planning to payment realization). This application is in the role of intermediary between the Treasury, organization distributing state budget resources, and individual non-profit organizations that are allowed to spend relevant part of these resources.   

Clients, whose virtual or real bank accounts are opened in the Treasury, use QBSW Expenditure Management as a internet banking in order to define spending purposes and sending payments to recipients for authorized purposes. The Treasury Single Account system allows to use such internet banking system also for execution of extra-budgetary funds of clients.

Page 49: mesmerizers.files.wordpress.com  · Web viewQ. Explain concept of Balance Scorecard. Ans: The Balanced Scorecard (BSC) is a performance management tool which began as a concept for

Main functions Revenues and expenditures at all levels of defined classifications Recording and adjusting plans (parameters) of future expenditures and revenues Approving plans according to funds availability Recording and approving commitment entry requests according to funds availability and presented

documentary proof Recording and approving requests for payment (payment orders) according to funds availability

and presented documentary proof Access to predefined special operations Execution of extra-budgetary funds Providing communication with the Treasury – short notices, free-format requests Generating reports for all transactions supported by the system

Page 50: mesmerizers.files.wordpress.com  · Web viewQ. Explain concept of Balance Scorecard. Ans: The Balanced Scorecard (BSC) is a performance management tool which began as a concept for

Q. What is ‘Two-step transfer pricing and profit sharing’ approach? Narrate merits & demerits thereof.Ans: Two – step pricing ApproachIt includes 2 charges:First, for each unit sold, a charge is made that is equal to the standard variable cost of production. Second, a periodic (usually monthly) charge is made that is equal to the fixed cost associated with the facilities reserved for the buying unit. One or both of these components should include a profit margin.

Profit Sharing ApproachIt is used to ensure congruence between business unit and company interest. This system operates as follows:1. The product is transferred to the marketing unit at standard variable cost.2. After the product is sold, the business units share the contribution earned, which is the selling price

minus the variable manufacturing and marketing costs.This method of pricing may be app. If demand for the manufactured product is not steady enough to warrant the permanent assignment of facilities, as in the 2step pricing method. In general, this method does make the marketing unit’s interest congruent with the company’s.

Q. Adopting Profit Center Approach may not be appropriate solution always`. Do you agree with this statement? Give reason for your answer quoting various situations in business.Ans: Establishing organization units as profit centers provides the following advantages:

The quality of decisions may improve because they are being made by managers closest to the point of decision.

The speed of operating decision may be increased since the do not have to be referred to corporate headquarters.

Headquarters management, relieved of day-to-day decision making, can concentrate on broader issues.

Managers, subject to fewer corporate restraints, are freer to use their imagination and initiative. Because profit centers are similar to independent companies, they provide an excellent training

ground for general management. Their managers gain experience in managing all functional areas, and upper management gains the opportunity to evaluate their potential for higher-level jobs.

Profit consciousness is enhanced since managers who are responsible for profits will constantly seek ways to increase them. (A manager responsible for marketing activities, for example, will tend to authorize promotion expenditures that increase sales, whereas a manger responsible for profits will be motivated to make promotion expenditures that increase profits.)

Profit center provide top management with ready-made information on profitability of the company’s individual components.

Because their output is so readily measured, profit centers are particularly responsive to pressures to improve their competitive performance.

EXAMPLE. ABB (Asea Brown Boveri), a European multinational in the business of power generation, transmission, and distribution, was organized into 4,500 small profit centers-each with profit and loss

Page 51: mesmerizers.files.wordpress.com  · Web viewQ. Explain concept of Balance Scorecard. Ans: The Balanced Scorecard (BSC) is a performance management tool which began as a concept for

responsibility and meaningful autonomy. Percy Barnevik, ABB`s CEO, explained why: “We are fervent believers in decentralization. When we structure local operations, we always push to create separate legal entities. Separate a company allows you to create real balance sheets with real responsibility of cash flow and dividends. With real balance sheets, managers inherit results from year to year through changes in equity. Separate companies also create more effective tools to recruit and motivate managers. People can aspire to meaningful career ladders in companies shall enough to understand and to be committed to”.

Many Japanese companies use profit centre. The Kyocera Corporation, a technology company divided itself into 800 small companies (nick named amoebas), which were expected to trade both internally and externally. HIgashimaru Shoya, a soy sauce maker, turned each stage in production process into a separate profit centre, instructing these separate units to buy and sell to any other. Matsushita, consumer electronics giant, operated its divisions as profit centers and focus managers attention on two numbers-profit margin and the “bottom line”. The consumer electronic industries were characterized by two factors: product life cycles tended to be short and profit margins were higher in the initial stages of the product life cycle than in the later stages. The focus on “profit margins” motivated managers to introduce new products, and the focus on the “bottom line” motivated managers to extract the minimum profits from current products.

Difficulties with profit centersHowever, the creation of profit centers may cause difficulties like:

Decentralized decision making will force top management to rely more on management control reports than on personal knowledge of an operation, entailing some loss of control.

If headquarters management is more capable or better informed than the average profit centre manager, the quality of decision made at the unit level may be reduced.

Friction may increase because of argument over the appropriate transfer price, the assignment of common costs, and the credit for revenues that were formerly generated jointly by two or more business units working together.

Organization units that once cooperated as functional units may not be in competition with one another. An increase in profits for one manager may mean a decrease for another. In such situations, a manager may fail to refer sales leads to another business unit better qualified to pursue them; may hoard personnel or equipment that, from the overall company standpoint, will be better off used in another unit; or may make production decisions that have undesirable cost consequences for other units.

Divisionalization may impose additional cost because of the additional management, staff personal, and record keeping required, and may lead to task redundancies at each profit center.

Competent general managers may not exit in functional organization because there may not have been sufficient opportunities for them to develop general management competence.

There may be too much emphasis on short rum profitability at an expense of long run profitability. In the desire to report high current profits, the profit centre manager may skimp on R&D, training programs, or maintenance. This tendency is especially prevalent when the turnover of profit centre

Page 52: mesmerizers.files.wordpress.com  · Web viewQ. Explain concept of Balance Scorecard. Ans: The Balanced Scorecard (BSC) is a performance management tool which began as a concept for

managers is relatively high. In this circumstance, managers may have good reason to believe that their actions may not affect profitability until after they have moved to other jobs.

There is no completely satisfactory system for ensuring that optimizing the profits of each individual profit center will optimize the profits of the company as a whole.

Q.- Discuss and illustrate difference and similarities between:-(a) Strategy Formulation & Task Control:Ans.:-

Strategy Formulation Task Control

1. Acquire an unrelated Business 1. Coordinate order entry

2. Enter a new Business 2. Schedule production

3. Add direct mail selling 3. Book TV Commercial

4. Change Debt/Equity Ratio 4. Manage cash flows

5. Adopt affirmative action policy 5. Maintain personal records

6. Devise Inventory 6. Reorder an item

7. Speculation Policy 7. Run individual research project

8. Decide Magnitude & Direction of research

(b) Management Control & Task Control:

Management Control Task Control1. Introduce new Product or Brand within product

line1. Coordinate order entry

2. Expand a plant 2. Schedule production3. Determine advertising budget 3. Book TV Commercial4. Issue new debt 4. Manage cash flows5. Implement minority Recruitment Program 5. Maintain personal records6. Decide Inventory levels 6. Reorder an item7. Control Research organization 7. Run individual research project